MBE Contracts
Defenses 4. Undue Influence 5. Unconscionability
Rule/Undue Influence: An aggrieved party may avail himself of the defense of undue influence when the circumstances reveal a vulnerable, but not incapacitated, party who succumbs to untoward bargaining tactics and pressures from the other party, where the those pressures or tactics fall short of fraud or duress. There are two elements to a defense of undue influence: (1) unfair persuasion was used -(a) although the test for unfair persuasion is one that looks at all of the circumstances, courts have looked in particular to the following indicia of untoward pressure: (1) discussion of the transaction at an unusual or inappropriate time; (2) consummation of the transaction at an unusual place; (3) insistent demands that the transaction or business be furnished immediately; (4) extreme emphasis on the untoward consequence of delaying the transaction; (5) the use of multiple persuaders against the target of persuasion; (6) absence of third-party advisers to the target of persuasion; and (7) statements that there is no time to consult financial advisors or attorneys; and (2) the other party was vulnerable to such persuasion - (a) A vulnerable party can be established in the following ways: (1) where the mental infirmity is due to age or illness but falls short of mental incompetence; (2) where the vulnerability is due to some recent trauma or event; (3) where the vulnerable party is reliant on the other party because of some relationship of trust or confidence with the other party. Rule/Undue Influence/Remedies: (a) Contractual obligations assumed under undue influence are voidable at the option of the aggrieved party; (b) the aggrieved party is entitled to restitution of any benefits conferred on the other party, but must return the value in excess of those benefits to the other party; (c) where the vulnerable party suffers undue influence at the hands of a third party rather than the other party to the contract, the aggrieved party has the power to avoid the contract at his option, unless two conditions are met: (1) the other party gives vales or relies materially on the transaction; and (2) the other party is proceeding in good faith without reason to know of the undue influence. Rule/Undue Influence/UCC: Like the other defenses, the common law claim of undue influence is available in cases under the UCC as a supplementary provision. Rule/Unconscionability: Just as with undue influence, the defense of unconscionability may be available when a party uses inappropriate bargaining tactics to take unfair advantage of a vulnerable party. Unconscionability, however, focuses, not only on the unfair process but also on the unfair results. There are two elements to a defense of unconscionability. In most jurisdictions, both of the following elements must be proved in order to succeed on a defense of unconscionability. However, where there is an extremely strong showing of one element, the party may be able to succeed on a defense of unconscionability despite a weaker showing of the other element. The elements are: (1) procedural unconscionability - this element can be met where the bargaining process that produced the contract in question created an absence of meaningful choice for the aggrieved party. The following are circumstances that may establish the absence of a meaningful choice: (a) near-miss cases [e.g., (1) when a party is vulnerable to the pressure of the other party, such as because of language barriers or advanced age, but the vulnerability falls short of mental incompetence; (2) when the victim is subjected to misleading or coercive sales tactics that fall short of actual fraud or duress]; (b)absence of bargaining power [e.g., (1) when the party is forced to accept proposed terms, as may be the case in standard form contract with "take it or leave it" provisions which sellers and suppliers impose on all customers and that cannot be varied by bargaining, greatly limiting the other party's power to bargain; (2) when one party is weakened by poverty or language barriers, which greatly limits his ability to shop for alternative terms particularly with respect to necessaries]; and (c) fine-print terms [e.g., where the contract included complex or arcane terms hidden in the maze of the fine print in a lengthy standard from contract]; and (2) substantive unconscionability - (a) the second element of unconscionability is met when the contract terms are unreasonably unfavorable to the aggrieved party. (b) Typical examples include: (1) grossly excessive price [e.g., where a pay-over-time plan requires the consumer to pay a total sum that is many time the value of the purchased goods, or where a bank charges an overdraft fee that is many times the bank's actual processing costs] ; (2) grossly disproportionate consequences for a minor breach [An add-on clause, pursuant to which a merchant is entitled to repossess multiple household items, when a consumer missed a payment, despite the fact that the consumer had already paid nearly 80% of the monies owed for the various purchases]; (3) provisions binding one party but not the other [An arbitration provision in an employment contract that requires the employee but not the employer to arbitrate any disputes, or that permits the employer but not the employee to amend the provisions of the agreement at will]; and (4) provisions which are grossly unfair [e.g., a binding arbitration clause that requires consumers to pay a non-refundable advance fee that exceeded the purchase price of the goods, travel to a distant city for the proceeding, pay the merchants legal fees in the event of a loss, and file all correspondence in a foreign county, Brower v. Gateway 2000, 676 N.Y.S.2d 569 (N.Y. App. Div. 1998)]. Rule/Unconscionability/Remedies: (a) upon a finding of unconscionability the court may: (1) refuse to enforce the contract; (2) excise the offending clause and enforce the remainder of the contract; (3) limit the application of the offending clause as to avoid any unconscionable result. Rule/Unconscionability/Application: (a) the question of whether or not unconscionability is present is a question of law to be decided by the courts; (b) the question of whether a particular contract or clause is unconscionable is to be determined by the circumstances existing at the time of contract formation, not at the time the dispute arose. (c) A party claiming or resisting a claim of unconscionability is entitled to present evidence of the allegedly offending provision's commercial setting. as well as its purpose and effect. (1) To refuse a party the opportunity to present such evidence before granting summary judgment is a reversible error. Rule/Unconscionability/UCC: The defense of unconscionability is specifically recognized as defense under the UCC, and applies in the same manner as under the common law.
Remedies 1. Monetary Remedies for Breach of Contract 2. Equitable Remedies 3. Other Possible Remedies
1. Monetary Damages at Common Law Rule/Monetary Damages at Common Law/General: A party aggrieved by a breach of contract may be able to recover money damages calculated to protect that party's expectation interest, reliance interest, or restitutionary interest. A party may elect only one of these three remedies. Rule/Monetary Damages at Common Law/Expectation Interest: To compensate a party for his expectation interest, the court will calculate money damage designed to put the aggrieved party where he would have been if the other party had fully and properly performed. (Formula: Loss of Value of the breaching party's performance + any incidental and consequential damages - any payments received from the breaching party - any costs saved as a result of the breach = Expectation Damages). Rule/Monetary Damages at Common Law/Limitations of Expectation Damages: The aggrieved party may not be able to recover the full amount of the expectation damages in the following situations: (1) Where the cost of performance greatly exceeds the market value of performance: (a) cost of performance - is the cost that would be incurred in an effort to perform as promised under the contract; (b) Market Value Performance - is the net increase in the market value of finding substitute performance of the contract [e.g., Peevyhouse v. Garland Coal & Mining Co., Because the restoration work would have cost the mining company $29,000 but would only increase the market value of the land by $300, the court limited the landowner's recovery to the latter figure]; (2) Where expectation damages cannot be calculated with reasonable certainty: (a) while an aggrieved party does not have to calculate with mathematical precision to receive expectation damages, there must be some reasonable basis for calculation; (b) doubts in calculations will be resolved in favor of the aggrieved party; (c) requiring reasonable certainty of expectation damages may leave the aggrieved party uncompensated for losses that cannot be easily valued (e.g., A new business is unable to open because of a breach by a key supplier who has not completed promised work. Because there is no proven track record for the new business, it is difficult to establish with any certainty what profits would have been realized had it opened on time); (d) some jurisdictions apply the "new business" rule and automatically bar recovery, but most jurisdictions will evaluate the facts on a case-by-case basis and award expectation damages if a reasonable calculation is possible (e.g., on the basis of profit records for similarly situated ventures) ; (3) where damages are unforeseeable (a) under the Hadley rule, a breaching party will be liable for general damages (those damages that naturally flow from the breach) but not for special damages, which are those damages that result from the particular circumstances of the aggrieved party unless, at the time of contracting, the breaching party knew or had reason to know that the consequential damages would result from the breach. A common type of consequential damages is lost profits; (b) the Hadley rule operates as another limitation on the right of an aggrieved party to recover expectation damages by denying recovery to the aggrieved party for unforeseeable consequential damages, even though those damages would not have occurred if the breaching party had fully performed on the contract; (c) the rule barring recovery for unforeseeable consequential damages is a default rule, and the parties may opt out of its application to their contract and instead allow liability for all consequential damages whether foreseeable or unforeseeable; and (4) where damages can be mitigated: (a) the aggrieved party may not recover for any losses it could have avoided without unreasonable risk, burden, or humiliation; (b) the aggrieved party's duty to mitigate damage is limited to taking reasonable efforts to mitigate. If such reasonable mitigation efforts fail or cause additional expense, then the breaching party will be fully liable [e.g., (1) the liability of a breaching buyer of produce would be reduced if seller permitted the produce to rot after breach rather than selling it to third parties; but if the seller is forced by the breach into a "fire sale" because of the short shelf life of the produce in question. then the breaching buyer will be fully liable for expectation damages; (2) the liability of an employer is reduced if an employee dismissed in breach of contract takes an extended vacation rather than seeking other work. But the aggrieved employee need only make reasonable efforts to secure a position that is reasonably equivalent to the job lost and, accordingly, she need not accept the substitute work when it: is in a different field; offers significantly lower pay or less desirable terms and conditions of employment than those of the lost job; would entail more burdensome responsibilities than those of the lost job; or would damage the aggrieved party's career prospect (e.g., an A-list star taking a lesser role in a B-grade film); (3) the liability of a breaching landowner to a construction contract hired to erect a structure on the property is reduced where the contractor continues the building project after the landowner has repudiated the contract. But the liability of a buyer of goods ordered for special manufacture would not be reduced if the seller continues the manufacturing process after breach in order to finish the goods for possible sale to third parties.) Rule/Monetary Damages at Common Law/Reliance Interest: A court will calculate the money damages designed to put the aggrieved party in the position he was before the contract was made. [e.g., A agrees to buy B's car. On the faith of the promise, A pays B $2000 down payment for the car and enters a 12-month lease agreement with C for a parking space for $2,400. The reliance interest of A would be $4,400. the amount it would take to compensate A for what he's lost on the faith of B's promise]. (1) Reliance damages may be available where expectation damages are not available; the reliance interest is compensated by the value lost the breaching party, as well as any other value lost from the aggrieved party's reliance on the breaching party. (2) The aggrieved party's reliance interest is measured by any expenditures made in preparation for performance or in actually performing, less any loss which the aggrieved party can prove, with reasonable certainty, that the aggrieved party would have suffered even if the contract had been fully performed; (3) the most common use of reliance damages arises when the expectation would be uncertain or speculative, as where the breach would deprive the aggrieved party of opportunities to enter potential transactions with third parties [e.g., Inventor signs a contract with railroad company for shipment of a new stove inventor plans to display at a manufacturer's convention, and he pays fees for the convention hotel for a room and for display space, as well as an exhibition fee to the convention sponsor. if the railroad company breaches the contract by failing to deliver the stove until after the convention, Inventor will be unable to recover damages for opportunities he may have lost due to his inability to showcase the stove, as calculating the probability and value of those opportunities would be too speculative. However, inventor would be entitled to reliance damages (i.e., the return of railroad co.'s fee, plus compensation for the amounts he paid to the hotel and convention sponsor). Rule/Monetary Damages at Common Law/Restitution: To protect an aggrieved party's restitutionary interest, a court will calculate money damages designed to return to the aggrieved party the value of the benefit he conferred to the breaching party. (1) a party aggrieved by a breach of contract is entitled to restitutionary damages rather than expectation damages at his option. If chosen, the aggrieved party may choose to recover the value of the benefits conferred on the breaching party by the aggrieved party during the course of the contract. (a) Restitutionary damages are available to both the aggrieved and the breaching parties. (2) The aggrieved party's restitutionary interest will be measured by either: (a) the reasonable value of the benefit conferred upon the breaching party. (i) this is measured by the market value of the services rendered, and not the price established by the parties in the contract; (ii) the contract price may be admissible in calculating the restitutionary damages, as evidence of the price of retaining those services on the market; or (b) the extent to which the breaching party's property has increased in value based upon the aggrieved party's performance [e.g., Contractor built a Turkish bathhouse on Homeowner's property. The market value of the services is the cost to Homeowner of hiring another contractor to do the work, which equals $30,000. The value added to Homeowner's property is $20,000. A court calculating Contractor's restitutionary interest could award him either amount, considering what relevant precedents permit and what justice requires}. (3) An aggrieved party is likely to make this election if the restitutionary recovery would exceed the amount recoverable based on his expectation interest, and situation is most likely to arise in the context of a "losing contract" (i.e., where the expectation interest would be less than zero because the aggrieved party would have actually lost money had the other party not breached; e.g., GC breaches it's contract with SC is halfway finished with its work under the subcontract. SC is entitled to seek from the breaching GC either expectation damages or restitutionary damages, i.e., the market value of the services already rendered at the time of the breach; even if subcontractor had a losing contract, it is nevertheless entitled to recovery of its restitutionary interest). (4) In order to secure restitutionary damages, the party seeking the remedy must have conferred some benefit on the other party through either part performance or reliance. (a) if the aggrieved party has fully performed under the contract then the aggrieved party is limited to expectation damages [e.g., SC completes contractual performance and GC refuses to pay. SC has no right to seek restitutionary damages and is entitled to recovery of the contract price only]. Restitution Reliance Recovery/Rule: An express promise by a debtor to pay all or part of an antecedent debt barred by the SOL or by a BK decree is legally enforceable without new consideration. The majority of jurisdictions requires that a promise or acknowledgement of a debt barred by the SOL must be in writing and signed by the debtor, only a few states require the same formality as to the express promise to pay the debt barred by SOL or discharged by BK. The creditor will have no right to collect on the debt until the time has lapsed expressly promised by the debtor. E. Liquidated Damages Provisions (a) Rule: Parties are free to include among the terms of their contract a liquidated damages clause designed to provide for damages of their own choosing in the event of breach. Such provision is enforceable if the court finds it to be a valid liquidated damages clause, and unenforceable if the court finds that it constitutes a penalty. A party has right either to perform on a contract or pay damages, and a penalty clause is seen as interfering with that right. (b) The test for determining whether a clause in a particular contract is a valid liquidated damages provision is three prongs: (1) Did the parties intend for the clause to operate as liquidated damages clause or as a penalty - (i) evidence of specific camouflaged effort to punish breach would support a claim that the clause was an intended penalty; sat (ii) modern decisions downplay the importance of this prongs and emphasize the other two. (2) Was the clause reasonable at the time of contracting in relationship to the anticipated harm - the key question is whether there was an anticipated harm that would be difficult to prove or for which no adequate remedy would be available. (3) Was the clause reasonable in relation to the harm and losses that actually occurred due to the breach? Liquidated Damages/Anticipated vs. Actual Harm 1. Rule: When a particular clause satisfies the second prong of the test (it was reasonable at the time of contracting in relation to anticipated harms) but not the third prong of the test (it wasn't reasonable in relation to the harms actually suffered), some courts will uphold the challenged clause as a valid liquidation of damages with the view that a clause that satisfies either one of the prongs will be valid. 2. Other court, however, will conclude such a clause a penalty if it specifies damages are grossly disproportionate to the actual harm, even if the clause was reasonable at the time of contracting. 3. The party attempting to show that the liquidated damages clause is actually unenforceable penalty has the burden of proof. If the party fails to offer evidence of the unreasonableness of the provision, then the liquidated damages provision will be upheld. 4. If the court concludes that the liquidated damages clause is, in fact, a penalty, then the clause will be stricken from the contract. If the clause is stricken from the contract, the aggrieved party is still entitled to recover whatever legal or equitable relief it would be entitled to under the law.
Performance, Modification, and Excuse 5. Conditions e. Rules Governing the Failure of a Condition
A. Rules Governing the Failure of a Condition Rule/Default: The default rules for a failed condition of the contract performance are dependent on whether the contract falls under the common law or under the UCC. Rule/Cases Governed by the Common Law: In common law cases, the rights of the parties in the event of a failed condition depend on whether the condition is either express or implied. (a) Express Conditions: where a party's performance under the contract is subject to an express condition, the failure of that condition will discharge the party's obligation to perform. (b) Identifying Express Conditions: (1) In some case, the parties will used magic words to describe a particular performance or event as a condition on a return promise; (2) the term "expressly conditioned" does not have to be included, but language conveying the same idea will constitute and express condition as well; (3) in the absence of clear language, an express condition may be established by trade usage, course of dealing, or course of performance evidence. Rule/Excusing Failed Express Conditions: (1) Generally, a failure of the occurrence of an express condition will discharge the performance obligation of the party who stood benefit from the condition; (2) nevertheless, there are three situations in which the failure of a condition may be excused without discharging the beneficiary's performance obligation: (a) Waiver- (i) the party who has been discharged from performing by the failed condition may waive the right to discharge and perform anyway. When the party waives the condition, the waiving party's obligation becomes absolute because it is no longer subject to the condition; (ii) some courts treat a waiver of condition as a midterm modification to the contract and require additional consideration be given by the other party to give the waiver effect. (b) Bad-faith Conduct-(i) a condition will be excused on the basis of bad faith conduct by the beneficiary of the condition. Thus, bad faith conduct will excuse the condition where the beneficiary party interferes with the fulfillment of a contract, or where the benefiting party fails to take steps necessary to the fulfillment of the condition. and (c) Avoiding Forfeiture: (1) in some situations the fulfillment of a condition may result in a forfeiture or a great loss to one of the parties. In those cases, the court may excuse the condition to avoid forfeiture; (2) In deciding to excuse those conditions the court will consider: (i) whether the party favoring excuse will suffer a loss greatly disproportionate to the actual prejudice to the other party; (ii) whether the failure of the condition is due to willfulness or serious neglect; (iii) whether the other party played a role in bringing that failure about; (iv) whether the condition relates to a minor term in the contract as opposed to a material one; and (iv) whether the fulfillment of the condition has not failed completely but has been merely delayed. [Note: There is another consideration that applies to insurance contract only. In deciding to excuse a condition, courts will consider whether, where a contract for insurance benefits conditions benefits payments on timely notice of covered events, such notice is not provided because of circumstances outside the insurer's control. (e.g., a failure to provide notice of a covered disability, where the disability itself has caused the failure of a timely notice, would be a condition that would be excused) or (failure to provide timely proof of death, where the body is not discovered for an extended period of time, would be an excused condition).] B. Implied Conditions (1) Rule: When the breach in question relates to an express condition, courts will enforce the condition to the letter and discharge the conditioned performance obligation, absent an excuse such as waived by bad faith. (2) Rule: When the possibility of breach is not addressed by an express condition, it is still a breach; HOWEVER, under the law of implied conditions, courts can treat the breach in one of two ways: either as (a) material breach or as (b) substantial performance. (3) Material Breach vs. Substantial Performance: (a) Rule/Material Breach: If the breach is serious enough, the court will treat the breach in the same way it would treat a breach of an express condition (i.e., as relieving the aggrieved party of how own performance obligation). This is called material breach, and the aggrieved party is free to walk away. [e.g., think of a contract to build a home using all copper pipes but instead subcontractor uses all steel pipes unbeknownst to the GC; this is a breach but GC substantially performed because landowner deprived only de mimimis benefit since the substituted pipe is functional and contractor would suffer tremendous loss to remove the pipe ; if steel pipes were used to cut costs and Owner allergic to steel this is a material breach because it would deprive the owner of a substantial benefit for which the contract was entered into] (b) Rule/Substantial performance: If the breach is less serious, the court will treat the performance as "close enough," meaning that the party has rendered substantial performance of the condition. In these cases, the aggrieved party will not be discharged of his performance obligation. The doctrine of substantial performance applies to construction contracts and contracts for services. Even where the court finds for substantial performance of a condition, the aggrieved party may still sue for damages as remedy for breach. (c) Rule: The Restatement of Contracts lists five factors that can help distinguish between material breach and substantial performance: 1. The extent to which the aggrieved party will be deprived of a benefit, which is reasonably expected under the terms of the contract; 2. The extent to which the aggrieved party can adequately be compensated via damages for the defective performance; 3. The extent to which the breach was willful or in bad faith, rather than merely negligent or innocent; 4. The extent to which the breaching party will suffer forfeiture if a material breach is found; and 5. The likelihood that the breaching party will cure his failure within a reasonable time and in manner consistent with the reasonable purpose of the contract [this is a very significant factor]. (d) Total Breach vs. Partial Breach Rule: A material breach can be treated as either a partial breach or a total breach. A claim for damages for total breach is one for damages based on all of the injured party's remaining rights to performance. A claim for damages for partial breach is one for damages based on only part of the injured party's remaining rights to performance. HOWEVER, a non-material breach can only be a partial breach. (e) Failed Condition that Cannot be Excused Rule: Where a condition has failed and cannot be excused, there are other methods of enforcement available to mitigate the consequence for the breaching party. If the contract requires both parties to render a series of performances over a period of time, the contract may be divisible in the event of a failed condition: (1) the failed condition must not be material to the contract itself, but must relate to a small portion of the contract performance in order for the contract to be severable to recover for portions properly performed. (2)The legal test for determining whether a contract is divisible is: (a) can the performance in question be apportioned into corresponding pairs of parties' performances? (b) can each pair be properly regarded as equivalents? (c) Where a party has failed to fulfill an express condition or is in material breach of he contract, he may still be able to recover in quantum meruit the value of benefits he has conferred to the aggrieved party during the performance: (1) the breaching party may recover the reasonable value of the benefits conferred under a theory of quantum meruit; (2) the reasonable value of he benefits conferred will be reduced by the damages suffered by the aggrieved party due to the breach. Rule: A helpful way to look at divisibility is by asking whether the contract is nothing more than the sum of its parts. If so, then the contract is divisible. If the contract involves more than the sum of its parts, then it is unlikely to be divisible. C. Sales of Goods Contracts under the UCC Rule/Perfect Tender Rule: the terms of a contract for the sale of goods are enforced exactly. Every contract term is thus treated as an express condition, and a breach of the performance obligation by the seller will relieve the payment obligation of the buyer. As a result there is no need to distinguish between express and implied conditions because they will be treated in the same fashion. Rule/Perfect Tender Rule: A buyer is enabled to reject a seller's goods "if goods or the tender of delivery fail in any respect to conform to the contract." Under this rule, any deviation from the performance specified by the contract (e.g., regarding the quality or quantity of the goods, or the time or manner of delivery) constitutes a breach by the seller. Rule/Perfect Tender Rule: If the seller fails to make perfect tender, the buyer has three courses of action available: (1) reject the goods; (2) accept the goods; or (3) reject part and accept part. (1) reject the goods: for a buyer to reject the goods, the buyer must exercise the right of rejection within a reasonable time after delivery and notify the seller of the rejection within a reasonable time period. (b) Rule: If the buyer rejects the goods, the buyer may bring an action for damages against the seller for imperfect tender, and the buyer must use reasonable care with respect to the holding of goods for a time sufficient for the seller to remove them. (i) if the seller gives no instructions within a reasonable time after notification of a rejection, the buyer may store the goods for the seller's account, reship them to the seller, or resell them for the seller's account. (ii) HOWEVER, if the buyer is a merchant and the seller has no agent or place of business in the place where the goods were rejected, the buyer must follow reasonable instructions received from the seller, and if none, must take reasonable efforts to sell the goods if they are perishable or their value will decline quickly. (iii) A merchant buyer who resells the goods after a rightful rejection is entitled to reimbursement for the reasonable expenses of caring for and selling the goods. (c) Rule: If the buyer fails to state in connection with the rejection a particular defect, the buyer will be precluded from relying on the unstated defect to justify rejection of the defect is ascertainable by reasonable inspection, where: (1) the seller could have cured the defect if it was stated seasonably; or (2) between merchants, the seller after rejection made a written full and final statement of all defects on which the buyer proposes to rely. (d) Rule: If a buyer does not effectuate rejection in the manner specified above, then he has made a failed rejection. A failed rejection will be deemed to be acceptance of the goods by the buyer. (2) Accept the Goods (a) Rule: The buyer may accept the goods, despite the improper tender. Acceptance occurs when the buyer has not had a reasonable opportunity to inspect the goods and signifies acceptance either through: (i) stating to the seller that the goods conform to the contract; (ii) taking the goods despite their non-conformance; (iii) failing to make effective rejection of the goods; (iv) taking any action that would be inconsistent with the seller's ownership of the goods (e.g., Buyer put the goods on display in the showroom) (b) Rule: If the buyer accepts the goods, he has the following rights and duties: (i) he must pay the contract price for the goods; (ii) he may seek damages against the seller for nonconformity if he notified the seller of the nonconformity within a reasonable time after discovering it, the seller is not prejudiced by the lack of notice, or his rights would not be affected; and (iii) he may revoke his acceptance if the nonconformity substantially impairs the value of goods, and he was initially unaware of the nonconformity because of the difficulty of discovery or because his acceptance was predicated on the seller's assurances of conformity or that nonconformity would be cured. (c) Rule: The buyer must revoke his acceptance within a reasonable time after the buyer discovers or should have discovered the grounds for revocation, and before there is any substantial change in the condition of the goods (unless caused by their own defect). The revocation is only effective once the buyer notifies the seller. 3. Reject Part and Accept Part of the Goods (a) Rule: A buyer who has received improper tender may accept some of the commercial tender and reject the rest. (b) Rule: The buyer will then have the same rights and duties of acceptance for the goods he accepted, and the rights and duties of rejection for the goods he rejected (though his picking and choosing is limited to "commercial units" - e.g., a buyer of bread can reject individual loaves but not half loaves). 4. Seller's Ability to Cure (a) Rule: If a seller makes a nonconforming tender but the time of performance has not yet expired under the contract, then the seller may substitute conforming goods so long as: (1) the seller gives the buyer seasonable notice of his intention to substitute; and (2) the seller makes conforming delivery within the time specified in the contact. (b) If a seller makes a nonconforming delivery and had reasonable grounds to believe that delivery would be acceptance to the buyer, the he may substitute a conforming delivery if: (1) the seller gives buyer reasonable notice of his intention to substitute; and (2) the seller make conforming delivery within a reasonable time. 5. Proof of Reasonable Grounds by Seller/Rule: A seller may prove that he had reasonable grounds to believe that the buyer would accept non-conformity if he has evidence of (1) express assurances to that effect; (2) trade usage, course of dealing, or course of performance evidence to that effect. 6. Special Rules for Installment Contracts: (a) Under the UCC, installment contracts are contracts that contemplate the delivery of goods in separate lots to be separately accepted by the buyer. 1. Rule: If the nonconforming installment substantially impairs the value of the whole contract, there is a breach of the whole contract; 2. Rule: If the nonconforming installment substantially impair the value of that installment, and the seller cannot cure, the buyer may reject that installment. HOWEVER, if the non-conforming installment does not substantially impair the value of the contract as a whole, and the seller gives adequate assurances that he will cure, the buyer must accept the installment.
Contract Formation: Offer and Acceptance 2. The Offer
A. The Offer Rule/The Offer: In order to constitute an offer, a party's communication must meet two requirements: (a) an outward manifestation (1) the manifestation can be oral, written, or mad via conduct; (2) inward thoughts or subjective intentions are irrelevant unless they are reasonably apparent to the other party [e.g.., (1) A seemingly serious offer to sell real property made in secret jest is nonetheless an offer; (2) A proposal to sell at a price that a reasonable person would regard as too good to be true (e.g., a new HDTV for $8.99) does not constitute an offer; and (b) the signal that acceptance would conclude the deal. (1) A offer must signal to the would-be offeree that the latter's agreement will conclude the deal. The key inquiry is whether the party making the communication expressed a willingness to commit without further assent [e.g., I will sell you my car if you'll pay me $2,000 cash." This is an offer because it expresses a willingness to conclude the deal if the other party pays $2,000] [e.g., "Yes, I'd be willing to sell you my car, but what are you willing to pay for it?" This is not an offer because the communicating party is obviously reserving the right to decide whether she likes the price suggested by the other party. She is thus withholding the privilege of further assent, or she is reserving the right to assent (or not assent) to the other party's proposal; (2) communications that withhold the privilege to further assent fall short of constituting an offer (a) a preliminary negotiation - is a generic term that applies to the give-and-take that occurs during bargaining [e.g., "We're willing to consider that price if you can be more flexible on the warranty and the payment terms]; (b) an invitation for an offer - is an advanced stage of preliminary negotiations, where the communicating party is closing in on a deal but wants the other party to commit first [e.g., "okay, we're in agreement on everything but price now. What's the best price you can give me?']. B. Multiple Offerees Rule/The Offer/Auctions: Auctions are situations where an item is sold to the highest bidder. The general rule is that the auctioneer is inviting offers, and the responsive bids are the offers. An auction is deemed to be with reserve unless specifically stated without reserve. EXCEPTION: Goods put up at an auction "without reserve" may not be withdrawn. Rule/The Offer/Advertisements: As a general rule advertisements for the sale of goods, circular letter, price lists, and articles displayed on a shelf with a price tag are construed as preliminary proposals inviting offers; because responses from recipients of the advertisement may exceed the available supply of goods or services; allocating the goods or services among responding parties will necessitate further assent from the advertising party. However in certain situations an advertisement for the sale of goods may constitute an offer [e.g., an advertisement in a newspaper proposing the sale of a coat "FIRST COME, FIRST SERVED" does in fact constitute an offer because the language in the ad indicates a quantity of one.] Identifying the means by which the goods or services will be allocated in the event of an excess demand, eliminates the need for further assent from the advertiser and, thus, constitutes an offer. Rule/The Offer/Reward Offers: A contract can only be formed if the offeree knew of the existence of the offer at the time of the alleged acceptance. There are two type of reward offers: (a) self-limiting rewards - are reward offers that indicate the task to be rewarded can possibly be performed only once. They are typically treated as offers that can only be accepted by the first person to render the requested performance in accordance with the terms of the offer. (b) Open-field rewards are reward offers that indicate that the task to be performed can potentially be performed by multiple parties. Unless otherwise specified in the offer, open-filed rewards can be accepted by any and all persons who render thee requested performance in accordance with terms in the reward. [e.g., The Carbolic Smoke Ball case, where the purveyor of a medical preparation promised to pay $100 to anyone who used the preparation as directed and, nonetheless, came down with the flu; (2) "Free Joe Six-Pack Action Figure! Send us your proof of purchase labels from six cases of your favorite Anheuser-Busch product with a self-addressed, stamped envelope. Offer good while supplies last! Because of the limitation stated in the offer, the power of acceptance exists only so long as supplies last. C. The Legal Effect of an Offer Rule/The Offer/The Legal Effect of an Offer: An offer creates the power of acceptance I an eligible offeree. This gives the offeree the power to create a contract simply by accepting the offer. When an eligible offeree exercises the power of acceptance before that power is terminated, a legally binding contract is formed between the parties. Generally there are 4 ways to terminate the power of acceptance; lapse of time, the death or incapacity of either party, revocation by the offeror, and rejection by the offeree. Rule/The Offer/The Legal Effect of an Offer/Lapse of Time: An offeree's power of acceptance terminates at the time stated in the offer. If the offer does specify the time of termination, then the power of acceptance will terminate after a reasonable time. The reasonable time determination is based on the following factors: (a) subject matter and market conditions; and what is considered a reasonable time will vary depending on the subject matter and the relevant market conditions [e.g., a reasonable time would be shorter for an offer to sell stock in a wildly fluctuating market than for an offer to sell in a more dormant market]; (b) the degree of urgency communicated by the means of transmission [e.g., a reasonable time would be shorter for an offer sent via e-mail than for an offer sent via postal mail]. Rule/The Offer/The Legal Effect of an Offer/Face-to-Face Communication: An offer made to one person to another in a face-to-face conversation is ordinarily deemed to remain open only until the close of the conversation. Rule/The Offer/The Legal Effect of an Offer/Death or Incapacity of Either Party: (a) the supervening death of either the offeror or offeree will terminate the power of acceptance with respect to the offer; (b) the supervening incapacity of either the offeror or the offeree, are evidenced by adjudication or the appointment of a guardian, will terminate the power of acceptance. Rule/The Offer/The Legal Effect of an Offer/Revocation by the Offeror: Under the rule of free revocability in American contract law, the offeror is free to revoke an outstanding offer, terminating the offeree's power of acceptance, at any time, for any reason, so long as the revocation: (1) occurs prior to acceptance; and (2) is effectively communicated. There are two methods of communicating a revocation: (1) Direct Revocation: typically occurs an terminates the offeree's power of acceptance when the offeror communicates directly with the offeree and advises the latter that the offer has been revoked. (2) Indirect Revocation: In some cases, the offeree will learn of the offeror's intention to abandon the deal from a third-party source, and this will also terminate the offeree's power of acceptance where the two conditions are met: (i) the offeror has take definite action inconsistent with the intention to enter the proposed contract (such as by selling offered real estate to a third party); and (ii) the offeree acquires reliable information of the offeror's inconsistent action (such as learning of the sale from a real estate broker). Rule/The Offer/The Legal Effect of an Offer/Revocation of an Offer Made to Multiple Offerees/Functional Equivalent Rule: Under the Second Restatement of Contracts, where an offer is made by advertisement in a newspaper or other general notification to the public or some segment thereof, the power of acceptance is terminated when the notice of revocation is communicated by advertisement or other general notification equivalent to that used for the offer and no better means of notification is reasonably available (the Functional Equivalent Rule). Option Contracts and Firm Offers Rule/The Offer/Option Contracts: An offeror is free to revoke any time before acceptance. Under the common law rule of [Dickerson v. Dodds (1876), the offeror can revoke even if he has expressly promised the offeree that he would hold the offer open. Under the modern law, there are two ways to prevent revocation of an offer: a common law option contract or a firm offer under the UCC. Rule/The Offer/Option Contract: Three elements are required for an enforceable option contract: (1) an offer; (2) a subsidiary promise to keep the offer open [a "sell-by" date is not necessarily a promise to keep the offer open. It can also constitute an express lapse provision, establishing a deadline for when the power of acceptance will terminate); and (3) some valid mechanism for securing enforcement of the subsidiary promise [the most common way is by giving consideration in return - it can be in the form of performance (e.g., payment option) or a promise of performance (e.g., where the promisee promises to use his "best efforts"). Rule/The Offer/Option Contract/Minority: A minority of jurisdictions have held that revocation of an offer can be prevented where the offer is in writing and signed by the offeror, recites purported consideration for the promise to keep the offer open, and proposes an exchange on fair terms within a reasonable time. Rule/The Offer/Option Contract/Promissory Estoppel: Under promissory estoppel, courts will sometimes enforce a subsidiary promise to keep an offer open where the offeree has foreseeably and reasonably relied on the option, and injustice can only be avoided by enforcing the promise [e.g., T on lease with option to purchase - the LL knew T remodeled the kitchen 6 mos before lease expires LL tells T, they are not renewing the lease; an option contract result because LL knew T was relying to their detriment]. Rule/The Offer/Option Contract/MBE: When a general contractor, about to submit a bid on a construction project, secures a bid from a subcontractor for a definite part of the proposed work, and uses the bid to determine that part of the costs, the subcontractor includes the general contractor's reliance on that bid. Because an offer which the offeror should reasonably expect to induce action or forbearance of a substantial character on the part of the offeree before acceptance and which does induce such action or forbearance is binding as an option contract to the extent necessary to avoid injustice. Rule/The Offer/Option Contract/Special Rules for Construction Projects: (a) A general contractor bidding on a construction project solicits bids from different subcontractors, such as, plumbers, electricians, or glazers. The general contractor must then use these bids in formulating his own bid, but cannot as a practical matter accept those bids unless and until the general contract is awarded. (b) The problem occurs when a subcontractor revokes his bid (offer) after the general contractor has relied on it to make his own bid on the primary contract, and the general contractor is then awarded the contract based on that bid. (c) The majority rule is that where a general contractor used a particular subcontractor's bid to formulate his own, an "implied option contract" is created via promissory estoppel. This prevents the subcontractor from revoking the bid despite the fact that the subcontractor hasn't promised to keep the bid and the general contractor hasn't provided any consideration to keep the bid open. Rule/The Offer/Option Contract/The UCC "Firm Offer:" Under the UCC, a merchant can make a firm offer (an irrevocable offer) to either by or sell goods without consideration so long as 2-205: (i) the offer is made by a merchant; (ii) the offer is made in writing signed by the merchant; and (iii) the offer expressly states by its terms that it will be held open. (b) The UCC defines a merchant in terms of his special knowledge or skill with respect to the practices or goods involved in a transaction. It is important to note that a person may be considered a merchant even if he only has knowledge of the goods, or knowledge of the practices. (c) A firm offer meets all of the above requirements becomes irrevocable either for the period of time sated in the firm offer or for a reasonable time if no time is specified; (d) Under the UCC, the shelf life of a firm offer can be no more than 3 mos. Any firm offers that a time longer than 3 mos. will only be irrevocable for the first 3 mos. The offer would then become revocable, but still enforceable. (e) A merchant selling or buying goods can establish an irrevocable offer for longer than the 3 mos. period stipulated in the UCC by creating a valid common law option contract. This means that the requirements of a subsidiary promise to keep the offer open, as well as some consideration in return, must also be present. Rejection by the Offeree Rule/The Offer/Rejection by the Offeree: (a) The power of acceptance can also be terminated if the offeree refuses to accept the offer. There are three ways that rejection can be affected: (1) outright rejection: the offeree's power to accept an offer terminated by his rejection of the offer. The offeree has the power to accept the offer, but the offeree cannot change his mind and try to accept the offer once he has rejected it; (2) rejection via counteroffer: (a) a counteroffer made on the same subject matter operates to simultaneously reject the initial offer; (b) not all statements or questions about an offer are considered counteroffers. An offeree may test the waters by making a "mere inquiry" about the offeror's willingness to negotiate without creating a counteroffer. An offeree's power of acceptance is terminated by the making of a counter-offer [Note: it is not the offeror's rejection of the counter-offer that terminates the offeree's power of acceptance but the fact that a counter-offer was made] (e.g., "I am not willing to pay $10,000 for the car, but I would happily buy your car for $9,000." This statement would rise to the level of a counter-offer because the terms of the initial offer, to purchase the care at $10,000, have been modified by the offeree to $9,000. As a result, the offeree made a counteroffer which at the same time rejected the offeror's initial offer; e.g., "10,000 is not out of the question, but it is a little high, given the age of the car. Would you be willing to consider a lower offer?" This language would not rise to the level of a valid counteroffer because the offeree is only asking if the offeror is willing to negotiate the terms, and not proposing a different total). (3) Rejection via nonconforming acceptance: (a) at common law, the mirage image rule requires acceptance must mirror the terms of the offer, and any variation results in a counteroffer and rejection of the initial offer. Despite the mirror image rule, a de minimis variation in the terms of the acceptance may constitute acceptance in some situations: (i) if a court concludes that the minor variation was implicit in the offer, or merely a suggestion for the addition of a new term, then the response may constitute acceptance mirroring the offer, creating a contract; (ii) if the purpose of raising the issue is to attempt in bad faith to back out of the contract, a de minimis variation in the terms of acceptance may still constitute acceptance. (b) The UCC rejects the common law mirror image rule and recognizes a binding contract despite the presence of nonconforming acceptance in two sets of circumstances: the shipment of nonconforming goods and the battle of the forms. Rule/The Offer/Revival of the Offer: The maker is the master of the offer. Therefore, an offeror has the power to revive an offer that the offeree has rejected, and with it the offeree's power of acceptance, and he can likewise revive an offer that has lapsed; all he must do is communicate the revival to the offeree. This can be accomplished by restating the offer or giving the offeree more time to make a decision. D. Offer and Acceptance Under Unilateral Contracts Rule/Offer and Acceptance Under Unilateral Contracts: An offer can require acceptance by either a promise or performance. Rule/Bilateral Contract: An offer seeking a promissory acceptance is an offer to enter into a bilateral contract, the offeree can accept the offer by making the requested promise. In this situation both parties are bound by the terms of the contract once the mutual promises are exchanged. Rule/Unilateral Contract: An offer seeking performance in return is an offer to enter into a unilateral contract. The offeror is bound only when the offeree completes performance in accordance with the terms of the offer; and the offeree is never bound to perform because he has never promised to perform. It is entirely up to him whether to perform or not. However, he will not be entitled to the benefits of the offeror's promise unless and until he renders the required performance. Where an offer does not specify whether it must be accepted by a promise or part performance, the offeree is free to choose the means of acceptance. However, the offeree must accept via performance where terms of the offer or surrounding circumstances make it clear that a performance is required. [Note: Reward Offers is a type of unilateral offer BUT the offeror will not settle for a promise and is instead seeking the actual performance before binding himself to the deal] ALSO[Rather than a promise to sell a house, most real estate brokerage agreements include a promise by the owner to pay a fee in exchange for the actual sale of the property, which would constitute performance]. Rule/Revocation of the Offer in a Unilateral Contract: Under common law, the offeror was free to revoke the unilateral offer up until the moment that the offeree actually completed performance. MODERN RULE: once the offeree begins performance, an option contract is created and the offeror may not revoke (Not Majority Rule). EXCEPTION: An offeror may revoke under the modern rule where the offeree is engaged in "mere preparation" to perform rather than beginning performance itself.. Rule: although the offeror cannot revoke once the offeree has begun performance, acceptance of the offer is still effective only upon completion of the performance. The task must be completed in accordance with the terms of the offer in order for the offeree to accept the offer and bind the offeror to his performance obligation. Rule: The offeree of a unilateral contract is free to abandon the performance at anytime and even to not undertake the performance at all. Rule: If an offer to enter a unilateral contract was mailed to the offeree, then the offeror may revoke even if the offeree has place the acceptance in the mail. The mailbox rule only applies to bilateral contracts, and so dispatch could not constitute acceptance to create a unilateral contract.
Service Seller's/Continuing Obligation under Warranties Issue: Whether the service seller has an ongoing continuing duty after the service fails?
Consider the whether the type of service is one that would require a continuing duty; consider warranty as basis for such a duty. If the seller reasonably had a duty, then it would be entitled to reasonable notice of problems and reasonable time to address them.
SOF 1. In General 2. The SOF in Operation 3. Evaluating SOF Issues
1. In General Rule: The general rule in contract law is that a contract need not be in writing and that oral and written agreements are equally enforceable. HOWEVER, the SOF provides an important exception to that rule and makes some classes of contracts unenforceable unless reflected into a signed writing. The SOF has been adopted in every U.S. jurisdiction through either the legislature or common law. The SOF can be divided into two general areas of application: (1) certain sales of goods [the SOF governs certain sales of goods in every jurisdiction except for Louisiana]; and (2) certain transactions not involving the sales of goods [other types of transactions do not all under 2-201 of the UCC may still be governed by the SOF where states adopted the original SOF or some modified version of it]. 2. The SOF in Operation Rule: The SOF applies to certain categories of contracts, such as contracts for the sale of real estate or contracts lasting more than one year. For these contracts, the SOF requires that the contract be evidenced in writing signed by the party against whom enforcement is sought. In the context of litigation, the SOF is a defense which, if available, may be raised against a party who has claimed breach of contract. The defense is available to the breaching party when the transaction in question falls within one of the categories governed by the SOF, and the required writing in absent or deficient. 3. Evaluating SOF Issues: Rule/General: The general rule is that a contract within the SOF is not enforceable absent a writing signed by the party against whom enforcement is sought. The most efficient way to analyze SOF problem is by analyzing three questions: (1) Does the agreement fall within the SOF? (a) if the agreement does not fall within SOF is necessary to proceed further with the analysis; (2) Is the SOF satisfied? (a) if the signed writing requirement is met the SOF is satisfied, then the SOF will not render the contract unenforceable. (b) if the contract is within the subject matter of the SOF but the SOF is not satisfied, then the party seeking to enforce the contract will lose; and (3) Is alternative enforcement available? (a) if the contract is within the SOF, but the SOF is not satisfied, the party seeking to enforce the contract will have to use another theory of enforcement, such as promissory estoppel or quasi-contract, in order to protect its interest. The final step in SOF analysis is to look for any of those alternative theories that could apply.
Performance, Modification, and Excuse 4. Excusing Performance by Agreement of the Parties 1. Rescission 2. Accord and Satisfaction 3. Anticipatory Repudiation 4. Bilateral Executory Accord 5. Novation
1. Rescission Rule: (a) The parties may also agree to discharge each other's remaining duties of performance under an existing contract that is at least partly executory on each side. Typically, consideration will be provided by each party's discharge of the other's duties. (b) The SOF will not prevent an oral agreement of rescission that discharges unperformed duties from being enforced unless rescission of a transfer of property is involved. 2. Accord and Satisfaction Rule: (a) The parties may make an accord, which is a contract under which the obligee promises to accept substituted performance in satisfaction of the obligor's existing duty. Satisfaction - performance of the accord - will discharge he original duty. (b) Accord alone will not discharge the original duty, but once the accord is made, the original duty will be suspended subject to the terms of the accord until the obligor has the chance to make the substituted performance (1) if there is such a breach of the accord by the obligor as to discharge the obligee's duty to accept the substituted performance in satisfaction, the obligee is no longer bound under the accord, and he may choose between enforcing the original duty or the duty under the accord; (2) if the obligee breaches the accord, the original duty is not discharged, but the obligor can seek specific performance of the accord (in addition to damages for partial breach). Rule/Validity of Accord - Consideration Required: (1) General contract law applies to accords, such that consideration is required; (2) there may be sufficient consideration if the substituted consideration differs significantly from that required by the original duty; or because the original duty was doubtful or the obligor believed it to be doubtful; (3) Generally, there may be an accord and satisfaction when a creditor agrees to accept part payment of an unliquidated debt that the debtor tenders in full satisfaction of the debt; however, in order for part payment to constitute sufficient consideration on its own, there must be a bona fide or good faith dispute as to the debt. 3. Anticipatory Repudiation: Rule: Anticipatory repudiation occurs when, prior to the time that performance is due under the contract, a party announces his intention not to perform, or circumstances make such an intention reasonably clear to the aggrieved party. Technically, anticipatory repudiation is not the same as a breach of contract because the party has not yet failed on its promise of performance. However, the aggrieved is given the option to immediately treat the anticipatory repudiation as a breach of contract to be entitled to the same rights and remedies. Anticipatory repudiation may be established by: (1) a party's definitive indicating that he will commit a breach of contract; or (2) a party's voluntary or affirmative act that renders the party unable to perform or apparently unable to perform. Rule/Adequate Assurances of Performance: If anticipatory repudiation cannot be established but there are reasonable grounds for insecurity (i.e., reasonable grounds to believe that the other party is unwilling or unable to perform), the insecure party may make a demand for adequate assurance of performance. Adequate assurance of performance must be provided in the following forms: (a) the UCC requires that a demand for adequate assurance of performance on a sale of goods contract be made in writing; and (b) under the Restatement of Contracts, and adequate assurance of performance may be either: (1) by oral communication; or (2) in writing. Rule/Suspension of Performance by the Insecure Party: (a) upon making a demand for assurances, a party with reasonable grounds for insecurity may suspend its own contractual performance so long as: (1) suspension is commercially reasonable; and (2) the insecure party has not yet received the agreed-upon return for the performance in question. (b) The insecure party may also suspend his own performance where the performing party: (1) does not respond to a demand for assurances within a reasonable time (30 days under the UCC); or (2) does not respond in a manner that provides reasonable assurances to the other party - in this situation, the reasonableness of an assurance of performance is determined by: (a) the circumstances; (b) the relationship between the parties; (c) the parties' past dealings; and (d) the nature of the insecurity requiring assurance. The failure to respond within reasonable assurances constitutes a repudiation of the contract by the non-responding party. Rule/Rights of the Aggrieved Party upon Repudiation: If the aggrieved party choses to treat the anticipatory breach as a breach of contract he may: (a) cancel the contract and terminate all rights and obligations under it; or (b) bring an action for damages under specific performance. The aggrieved party may immediately resort to one of these remedies upon repudiation, or he may wait until performance should have occurred. If an aggrieved party choses to ignore a repudiation, then he is prevented from continuing to perform on the contract if performance would increase his damages on the contract. If the aggrieved party can prove to the court that he is willing, ready, and able to render performance had the anticipatory repudiation not occurred, then the aggrieved party is relieved of: (a) performance of contractual obligations; and (b) performance of any conditions precedent. Rule/Retraction of Repudiation: A party who made an anticipatory repudiation to the other party may retract the repudiation UNLESS and UNTIL the other party: (a) acts in reliance on the repudiation; (b) positively accepts the repudiation by signifying this to the breaching party; or (c) commences a suit for damages or specific performance. 4. Bilateral Executory Accord Rule: A bilateral executory accord is an agreement that an existing claim shall be discharged in the future by the rendition of a substitute performance. [e.g., Creditor tells Debtor that he promises to discharge a debt if Debtor delivers to him his collector's baseball card. Debtor promises; their agreement is a bilateral accord; its performance is the satisfaction (Note: if Debtor delivers the baseball card and Creditor accepts it, there is an accord and satisfaction). 5. Novation: Rule: Novation is synonymous to a "substituted contract" usually involving at least obligor or obligee who was not a party to the original contract (Note: there must be a substituted party in the MBE question).
SOF 5. Analyzing Problems Under UCC SOF
1. Specific Coverage under the UCC Rule: All contracts for the sale of goods with a price of $500 or more must satisfy the SOF. With regard to agreements for which the price is payable in some other form other than money (e.g., goods), the monetary value of the consideration must be determined in order to apply the $500 rule. Rule/Multiple Goods: For agreements contemplating the sale of multiple goods, calculation of the price will depend on whether the transaction constitutes a single contract or multiple contracts. When an agreement falls within the UCC SOF and the SOF is not satisfied, the entirety of the contract is unenforceable, not just the portion the contract which exceeds $499 Rule/Modification of the Value of Goods: The UCC SOF also applies if a contract as modified falls within the SOF (e.g., the contract is modified such that the price of the goods has gone from $300 to $600]. By the same taken if the new contract falls outside of the SOF, it does not apply [UCC 2-209(3)]. 2. Satisfaction of the UCC-SOF Rule/Satisfaction of the UCC-SOF: There are five ways to satisfy the UCC SOF: (1) a writing; (2) merchant's confirmation; (3) In-Court Admission; (4) Part Performance; and (5) Substantial Reliance by the Seller of Specifically Manufactured Goods Rule/Satisfaction of the UCC-SOF/Signed Writing: The easiest way to satisfy the UCC SOF, a signed writing requires: (1) a writing: (a) although the UCC defines writing as any intentional reduction to tangible form, which clearly covers everything from quill and parchment to computer printouts, a more difficult question is presented by electronic documentation (e.g., e-mail). A majority of courts have concluded that electronic documentation satisfies the writing requirement under the UCC. (b) The proposed revisions to UCC Article 2 replace the word "writing' with the word "record" for the express purpose of covering electronic documentation in the UCC SOF and other provisions. (2) that is signed by the party against whom enforcement is sought. (a) The UCC broadly defines "signed" as any symbol executed or adopted by a party with present intention to authenticate the writing, which covers the same variety of signings effective under the common law SOF. (b) Most states have adopted the UETA, under which electronic signatures satisfy legal writing requirements. (3) Which is sufficient to indicate that a contract for sale has been made between the parties. (a) all that is required is that the writing afford a basis for believing that the offered oral evidence rests on a real transaction, a very relaxed standard. Rule/Satisfaction of the UCC-SOF/Quantity of Goods: Satisfaction of the UCC SOF requires the term indicating the quantity of goods sold in the transaction. (1) the contract is unenforceable beyond the quantity of goods shown in the writing, irrespective of the parties actual agreement. (2) If there is no quantity term the contract is altogether unenforceable, subject to two exceptions: (a) where other language in the writing provides an unambiguous basis for measuring the quantity; and (b) in the case of output and requirements contracts, the expression "output," "requirements," or other equivalent satisfies the quantity requirement under the UCC. (3) apart from the quantity term no other term is required in the writing. Moreover, it doesn't matter if a terms is incorrectly stated. However, a writing can have so few terms that it will no longer be sufficient to indicate that a contract for sale has been made between the parties [e.g., A and B enter an oral agreement under the terms of which A is to sell B five tobacco barns for a total of $5,000. After they shake hands, A takes out a piece of pater, writes "five barns," signs it, and gives it to B. If A subsequently backs out of the deal. hew will have a successful SOF defense against B despite the fact that he signed the writing with a quantity term. because the writing, taken as a whole, is insufficient to indicate that a contract for sale has been made between A and B. (c) Despite the fact that the riles under the UCC SOF are generally more easily satisfied than the common law SOF rules, a written offer that will satisfy the latter may not be enough to satisfy the UCC. (1) Because the UCC's writing requirement says that the writing must be sufficient to indicate that a contract for sale has been made between the parties. The plain language suggests that only a writing that is contemporaneous with or subsequent to a contract formation will satisfy the test. (2) HOWEVER, a written offer that proposes a sale of a goods at a price of $500 or more that meets the requirements of the UCC firm offer rule is enforceable against the signing merchant. In such a case, the UCC SOF is irrelevant, despite the fact that without documentary proof of acceptance, the offer is not sufficient to indicate that a contract for sale has actually been made. Rule/Satisfaction of the UCC-SOF/Merchants Confirmation: (a) The UCC SOF may be satisfied when two merchants enter an oral agreement and one of them send the other a written confirmation of the agreement. In such a circumstance, the SOF is satisfied against the recipient merchant if the latter fails to object to the confirmation in a timely fashion. (b) A valid merchant's confirmation requires a writing that (1) is sufficient against the sender (a) the question here is whether the confirmation is signed by the sender and contains a quantity term; (2) is in confirmation of the contract (the authorities are split on whether this language imposes a requirement on valid merchants' confirmations in addition to the requirement that the writings be sufficient against the sender. Some court hold that a writing that is sufficient against the sender is all that is necessary; other courts stress the "and: terms and require express language confirming the existence of a prior agreement [e.g., during a telephone conversation on March 8, A places an order and B agrees to fulfill it. A sends a signed letter to B that outlines the price, quantity, and delivery terms "per 3/8 discussion," and ends with the following language: "we look forward to you filling your order per these terms." Because the signed writing could be sufficient to satisfy the SOF against the sender A, some courts would hold that it would also be sufficient against the recipient. Other courts would require an express reference to the parties preexisting agreement or contract in order to satisfy the writing in confirmation of the contract. (3) is sent within a reasonable time of the making of an oral agreement; and (4) the contents of which the receiving merchant has reason to know. Rule/Satisfaction of the UCC-SOF/Merchant's Confirmation/Postal Misdirection: A confirmation that never reaches the intended recipient's place of business (e.g., because of postal misdirection) would not be valid against the recipient, since the latter would have no reason to know its content's. On the other hand, a confirmation that reaches the intended recipient's place of business but remains unopened through some neglect would meet the 'reason to know test." A confirmation that meets the foregoing requirements will satisfy the UCC SOF against the recipient unless the recipient provides written notice of objection to the confirmation's contents within 10 days of receipt, but a would-be objection that is itself sufficient against the objector, will satisfy the SOF against the objector. [e.g. (1) A and B enter an oral agreement under the terms of which A is to sell B five tobacco barns for a total of $5,000. The following day, B sends written confirmation of the agreement to A, and two days later B receives a signed reply that reads in pertinent part: "My understanding is that we don't have a contract between us, for as I mentioned, I am still entertaining other offers for the barns. My apologies for any confusion tat may have been caused." Because A's reply objects to the confirmation's contents, the confirmation does not satisfy the UCC SOF against A. (2) A's signed reply to B's confirmation reads: " I regret that I can't go through with the sale of my barns to you. Although $5,000 as you agreed to pay is very generous, my accountant has advised me of certain tax implications of the transaction that I hadn't considered, and I must therefore bow out." Because A's signed reply is itself sufficient to indicate that a contract for sale had been made between the parties - a contract from which A is not seeking to escape - the reply satisfies the UCC SOF against A. Rule/Satisfaction of the UCC-SOF/In-Court Admission: A third means of satisfying the UCC SOF occurs when a party against whom enforcement is sought "admits in his pleading, testimony, or otherwise in court that a contract for sale was made: (1) this exception applies whether the admission in question is voluntary (as in pleadings) or involuntary (as on cross-examination); (2) a number of courts have held that the existence of this exception requires that the SOF claims be resolved at trial rather than at the pleadings stage, since the party against whom enforcement is sought might be persuaded to admit to the existence of a contract in open court; (3) other courts have concluded that it would defeat the purpose of the SOF to permit a party attempting to enforce a contract to "badger the defendant through discover and trial simply because of the possibility of obtaining an admission. (4) the in-court admission exception will not secure enforcement of the contract in question beyond the quality of goods admitted. Rule/Satisfaction of the UCC-SOF/Part Performance: (a) despite the absence of a signed writing, an otherwise valid contract is enforceable with respect to goods for which payment has been made and accepted or which have been received and accepted. (b) the part performance exception does not apply unless there are actions by both parties indicating that a contract for sale exists. Thus, a payment can constitute par performance only if the buyer makes the payment and the seller accepts it, and delivery of the goods can constitute partial performance only if the seller makes the delivery and the buyer accepts the goods. (1) If a buyer pays cash, there is seldom any question whether payment has been made and accepted. Payment by check, however, is more contemplated, with some courts holding that receipt of a check without endorsement or negotiation does not constitute acceptance, and other courts holding that mere retention of the check for a reasonable period constitutes acceptance; (2) any act of dominion inconsistent with a seller's continued ownership of goods (e.g., moving them from the warehouse receiving are to a retail store) will constitute acceptance of the goods. (c) Divisible versus Indivisible goods: (1) if the contract in question involves divisible goods (e.g., widgets), part performance secures enforcement for any quantity that has already been paid for by the buyer or delivered by the seller; the contract is not enforceable beyond that quantity. (2) if the contract in question involves an indivisible good (e.g., a speedboat) the majority rules holds that partial payment secures enforcement of the entire contract, though a minority of courts refuses enforcement unless full payment has been made. Rule/Satisfaction of the UCC-SOF/Substantial Reliance by the Seller of Specially Manufactured Goods: (a) a seller is unlikely to be able to sell specially manufactured goods to other buyers if a would-be buyer backs out, thus leaving the seller with a potentially large reliance loss. Also, the fact of manufacture according to specifications attributable to the buyer provides strong evidence that an oral contract actually exists between the parties. (b) Accordingly, an aggrieved seller can secure enforcement of an oral contract by establishing the following elements: (1) the goods are to be specially manufactured for the buyer; (2) the goods are not suitable for sale to others in the ordinary course of business; (3) the seller has substantially begun to manufacture, or made commitments to procure, the goods; (4) the actions undertaken to begin to manufacture or procure occurred under circumstances that reasonably indicate that the goods are for the buyer; and (5) the actions undertaken begin to manufacture or procure occurred before the seller received notice of the buyer's revocation. Rule/Satisfaction of the UCC-SOF/One-year Provision: Authorities are divided on whether a party seeking enforcement of a sale of goods contract that is also within the common law one-year provision must satisfy the UCC and Common law SOF [e.g., merchants A and B enter an oral installment contract calling for A's delivery to B of 1,000 widgets per moth for two years at a price of $1.00 per widget. A sends B a written confirmation of the oral agreement that meets the requirements, to which B does not respond. B backs out of the deal, A sues for breach of contract, and B defends via SOF. Although the written confirmation clearly satisfies the UCC SOF, the contract will be nevertheless unenforceable in some jurisdictions because the agreement is also governed by the Common Law one-year provision. which is not satisfied because the party against whom enforcement is sought (B) did not sign the writing. Rule/Enforcement Where the UCC SOF is not Satisfied: (a) if a party has relied to its detriment on an oral contract whose enforcement is barred by the UCC SOF, the aggrieved party may be able to secure a remedy via promissory estoppel. (1) while some court hold that promissory estoppel is available in appropriate circumstances to secure enforcement of a promise otherwise barred by the SOF, other courts point to the express language of the UCC SOF "except as otherwise provided in this section, a contract is not enforceable, and conclude that it precludes promissory estoppel since the doctrine is not mentioned in 2-201; (2) among those courts that do recognize promissory estoppel, some impose enhanced proof requirements on the claimant, such as proving unconscionable injury or that the other party could be unjustly enriched without enforcement of the contract; (3) because of the unique vulnerability of construction contractors to revocation by their subcontractors, virtually all courts have protected a general contractor via promissory estoppel even if the oral subcontract in question is for goods at a price of $500 or more.
Defenses 1. Incapacity
A. Incapacity Rule/Incapacity/Infancy: Infancy is the time period before a person reaches the age of majority. An infant, commonly referred to as a minor, is any person who is under the age of 18. At common law, minors lacked the capacity to enter into a contract. The modern rule is that a minor may enter into a contract, but the contract is voidable at the option of the minor. HOWEVER, in certain circumstances the contract may be ratified, or alternatively, the minor may still be liable for the benefits received. EXCEPTION: In most states, minors enjoy the power of avoidance even if they are married or emancipated. HOWEVER, a minority of jurisdictions deprive married or emancipated minors the power of avoidance, making all contracts they enter into enforceable against them. Rule/Infancy/Power of Avoidance: (1) While minors have the capacity to enter a contract before they reach the age of majority, they also have the power to disaffirm contracts they enter into (power of avoidance), with some exceptions. (2) Power of avoidance means that the minor has the option of voiding the contract. HOWEVER, the contract is not void against the other parties to a contract, and so is enforceable against everyone but the minor. (a) There is an increasing trend toward limiting the minors' use of the power of avoidance to a defense only. The minor is allowed to disaffirm a contract that the other party seeks to enforce against him. HOWEVER, he may not use the defense to bring suit against the other party to secure restitution of monies already paid. (3) Upon exercising the right to disaffirm a contract, the minor is obligated to return to the other party goods received under the contract. (a) the minor must return goods if they are in his possession when he disaffirms the contract. HOWEVER, he is not liable for damage, wear and tear, or any other deprecation in value of the goods; (b) if the minor is not in possession of the goods at the time of disaffirming the contract because he sold the goods, then the minor will be obligated to turn over the proceeds of the sale of the goods to the other party; (c) a contract between a minor and another party may be for something that cannot be returned, such as a services contract or a lease. In such case, where the subject matter of the contract is non-returnable, the minor is under no further obligation to return or compensate the other party; (d) in a minority of jurisdictions, a minor can only disaffirm a contract by making the other party whole. In these jurisdictions, the minor would be liable for depreciation, as well as compensating the other party for the services or other non-returnable items. In addition to returning possession or the proceeds of a sale for returnable items. Rule/Infancy/Ratification: (1) For most contracts that minors enter into before reaching the age of majority, the minor may ratify that contract upon reaching the age of majority. (2) A minor may ratify most transactions entered into during infancy by making any manifestation to the other party of an intention to be bound by the original contract. (a) The minor's silence regarding the contract after reaching the age of majority is not sufficient to constitute a ratification unless the minor continues to take advantage of goods or services provided under the contract. (b) In a majority of jurisdictions, if a minor fails to disaffirm the contract within a reasonable time after reaching the age of majority, then the contract will be deemed ratified. (c) Once a minor disaffirms the contract, a majority of jurisdictions consider the contract void, with title to the property revesting in the other party (where possible). Once disaffirmed, a minor can no longer ratify the contract. Ratification must occur prior to the minor's avoidance of the contract. Rule/Infancy/Exceptions: (1) Necessaries - (a) A minor's contract for necessaries is voidable. HOWEVER, the other party has the right in quasi-contract to recover for the reasonable value of the goods or services provided. (b) Necessaries are those items considered necessary for survival. Most basic categories include things like food, shelter, clothing, and medical care; (i) In some jurisdictions, these items are only considered necessaries if provided to an emancipated minor whose parents cannot or will not provide for them; (ii) some courts are split on whether to include automobiles and/or education in the category of these necessaries; (c) the other party may secure the reasonable value of the goods or services only, not the contract price. Additionally, if the minor paid more than the reasonable value, then he is entitled to a refund of the difference between the amount paid and the reasonable value. (2) Misrepresentations by Minor - (a) In a majority of jurisdictions, if a minor has misrepresented his age to the contracting party in order to obtain the goods or services of the contract, he may be equitably estopped from proving his real age in court. This would deny the minor the defense of infancy and make the contract enforceable. B. Mental Incompetence Rule/Mental Incompetence: At common law, an insane person did not have the capacity to enter a contract, and any contracts he made were void. Under modern rules, the bar for establishing mental incompetence is lower - a party need not be insane to invoke the doctrine - ad the consequences of invocation depend on multiple factors. (1) The key in all cases is whether the person was incompetent at the time of contracting. (2) The party who becomes incompetent after entering a contract can escape contractual liability - if at all - via impossibility and related doctrines. Rule/Mental Incompetence/Requirements for Mental Incompetence: (1) If a party is adjudicated incompetent and a guardian is appointed, then this adjudication will be sufficient to establish mental incompetence for contract cases; (2) where there is no adjudication of mental incompetence by a court, then a party may still be declared mentally incompetent for the purposes of contract formation under the following circumstances: (a) Cognitive Defects - in almost every jurisdiction, a person will be deemed mentally incompetent and lacking capacity to enter a contract if the person is unable to understand in a reasonable manner the nature and consequences of the transaction [e.g., a person who is operating under the influence of delusions or hallucinations will be deemed mentally incompetent based on cognitive defects]; (b) Volitional Defects - in many jurisdictions, mental incompetence can be established if: (a) a person is unable to act in a reasonable manner in relation to the transaction; and (b) the other party has reason to know of this condition [e.g., a person who is a manic depressive would be able to avoid a contract if he was in a manic phase at the time of contract formation and if the other party had reason to know of that person's state based, for example, his erratic behavior]. Rule/Mental Incompetence/Legal Consequences: (1) In most jurisdictions, a contract entered into by an incompetent person is voidable. This means that the contract is enforceable at the option of the of the incompetent party, rather than void, which would make it unenforceable by either party. (a) Some jurisdictions, as well as the Second Restatement, hold that a person adjudicated incompetent under guardianship has no capacity to incur contractual duties, making such contracts void. (2) A party who is mentally incompetent at the time of contract may ratify the contract if he becomes competent at a later time. Ratification may be accomplished by an oral or written manifestation of the intention to be bound by the original contract. (3) If the mentally incompetent person exercises his power of avoidance and has received some benefit under the contract, then he is required to make the other party whole by paying the reasonable value of the goods or services rendered. (a) A mentally incompetent party will be responsible for damage, wear and tear, or any other depreciation in value of the goods. EXCEPTION: if the other party takes unfair advantage of the mentally incompetent person, and has knowledge of the person's incompetence, then the other party is entitled to a return "as is" of any goods still in the possession of the mentally incompetent party and will have no right to recovery for goods or services already consumed. (4) The rules governing the liability of a mentally incompetent party for contract for necessaries are the same as for minors. The mentally incompetent party will be liable to the other party for reasonable value of those goods and services. (a) For mentally incompetent persons, necessaries include the retention of legal services in connection with adjudicating mental incompetence.
Sources of Contract Law 2. The UCC
Rule/UCC/Sales & Goods: The UCC governs sales of goods. If the UCC would apply to a contract, then UCC provisions will trump any contract common law. (a) Sales: are any transactions in which the seller transfers title of goods to the buyer. (1) Leases, bailments, and any other types of transactions that involve goods are not considered sales; (b) Goods: are broadly defined under the UCC to mean any "movable item." (1) Examples of goods are ball bearings, Hummers, processed foods, and produce, the Mona Lisa, clothing, carpeting, furniture, and everything on shelves at Wal-Mart. The UCC also specifically defines the following as goods: growing crops, unborn animals, and identified things attached to realty, such as uncut timber or tobacco. (2) Intangibles (e.g., goodwill of a business), currency, stock, bonds, and other securities, the assignment of a legal claim, real property, services (e.g., construction and repair projects, landscaping, dry cleaning) are not goods. Rule/UCC/Contract Formation: Under UCC 2-204, a contract for the sale of goods may be made in any manner sufficient to show agreement, including conduct by both parties which recognizes the existence of such contract. Rule/UCC/Hybrid Cases: In hybrid cases which involve both the sale of goods and a services contract, the question arises as to which source of contract law would apply: (1) The majority rule is that the appropriate source is determined by the predominant purpose of the transaction: (a) to determine the predominant purpose of a contract, the following factors are used: (1) the language of the contract; (2) the nature of the supplier's business; of and (3) the relative value of the good versus the service. (2) The Minority rule: involves partitioning the contract, applying the UCC to the sale of the goods portion of the contract, and then applying the common law in the services portion of the contract [e.g., both the sale and installation of an audio system for an automobile and the sale of a car with a long-term service agreement involve goods and services]. Rule/UCC/Services Contract with Incidental Goods: A rendition of services with goods incidentally involved, such as a contract with an artists to paint a portrait, would be governed by the common law. Rule/UCC/Sale of Goods with Incidental Services: A sale of goods with services being incidentally involved, such as the sale of the installation of a water heater, would be governed by the UCC. Rule/UCC/Computer Software and Other Electronic Information: (1) The majority of courts treat computer software and other electronic information as goods governed by the UCC. (2) In Maryland and Virginia, the UCC has been displaced by the Uniform Computer Information Transactions Act for transactions involving electronic information. Rule/UCC/Sale of All Goods: The UCC governs the sale of all goods, not just the sale of goods priced at $500 or more. The UCC SOF [2-201] applies only to the sale of goods priced at $500 or more, requiring that a writing be signed by the party against whom enforcement of the contract is sought, but the rest of the UCC applies to the sale of all goods. Rule/UCC/Merchants: The UCC governs all goods, not just sales of goods involving merchants. The UCC has a number of provisions that create special rules for merchants [e.g., 2-205 - only a merchant can make a firm offer; 2-314 and 2-315 - the merchant warranty provisions]. Although the sale of goods by and between consumers is beyond the scope of the special merchant provisions, such sales are governed by all the other provisions of the UCC.
Contract Formation: Offer and Acceptance 3. Acceptance and the Common Law
Rule/Acceptance and the Common Law: There are only two general requirements to constitute effective acceptance: (a) under the mirror image rule, the acceptance must mirror the terms of the offer; (b) the acceptance must be communicated to the offeror. Rule/Acceptance and the Common Law/Communicating Acceptance: Once again, the maker is the master of his offer, and if the offer stipulates a particular means of communicating acceptance, then the offeree must utilize those means in order to make an effective acceptance. If the offer is silent as to the means of communication, then the offeree is free to use any reasonable means of transmission. Unless t is the circumstances indicate otherwise, a means of transmission is reasonable if it is: (a) the means used by the offeror; (b) the means customarily used in similar transaction; or (c) a means of communication that is equivalent in expeditiousness and reliability to the means used by the offeror. Rule/Acceptance and the Common Law/Communicating Acceptance/Requirement of Communication not Required: There are three instances where the requirement that acceptance be communicated to the offeror may not apply: (1) acceptance by silence - (a) generally, an offeree's silence in response to an offer cannot constitute acceptance except in the following three circumstances: (a) where the offeree takes the benefit of the offeror's services with a reasonable opportunity to reject them and with reason to know the offeror's intention; (b) where the offeror has given the offeree reason to understand that acceptance may be communicated by silence, in which case the offeree's silence will operate as acceptance if he intends as such [e.g., (1) An insurance company notifies A that his policy will be automatically renewed unless it hears otherwise from him; (2) same facts, except A does not intend to accept. In these circumstances, his silence will not operate as acceptance. This is a rare instance in which contractual liability depends on the subjective intentions of the party rather than his outward manifestation) ; and (c) where, because of previous dealings or other circumstances, it is reasonable that the offeree should notify the offeror if he does not intend to accept, in which case his silence will operate as acceptance [e.g., A newspaper sends a written notice that it continue delivering the paper after the subscription has expired unless it hears otherwise from A. When the subscription has previously been renewed in this manner without incident. A's silence in the face of notice will constitute acceptance]. (2) acceptance by performance - the maker is the master of the offer and, accordingly, the maker of an offer to enter a unilateral contract is free to make communication of acceptance a part of the required performance. But if he fails to do so, then acceptance is effective upon the offeree's completion of the requested performance whether or not the offer so notifies the offeror; and (3) acceptance by mail or other correspondence - (a) Under the common law mailbox rule, which is the rule in every American jurisdiction except the federal court of claims, acceptance by mail is effective upon dispatch so long as the acceptance is properly posted, with the correct address. (1) The mailbox rule applies only to acceptances and not to any other communication between the contracting parties. Thus, offers, revocations, rejections, and counteroffers are all effective only upon receipt by the other party; (2) once, the offeree dispatches his acceptance, he thereby creates a binding contract; (a) the offeror may not revoke an offer once acceptance has been dispatched [e.g., while B was considering A's offer, A had a change of heart and sent B a revocation. However, B dispatched his acceptance before receiving A's revocation. The parties have a contract because A's revocation was not effective until receipt by B and B's acceptance was effective upon dispatch, which was prior to his receipt of the would-be revocation] ; (b) once the offeree dispatches his acceptance, the parties have a binding contract and the offeree may not withdraw his acceptance [e.g., B dispatches her acceptance of A's offer and then has a change of heart, telephonically notifying A of the fact while the acceptances is still in transmission. The parties have a binding contract because B's acceptance was effective upon dispatch, and it is too late for either party to back out because a contract was formed); (c) the parties are bound even if the acceptance is lost in transmission and the offeror has no knowledge of that acceptance. Rule/Acceptance and the Common Law/Communicating Acceptance/Mailbox Rule other Means of Transmission: The mailbox rule applies to acceptances sent by any means of transmission which involves a foreseeable delay between the time of dispatch by the offeree and the time of receipt by the offeror. (a) this definition would include a private overnight delivery service; (b) this would not include an instantaneous means of communication, such as facsimile; (c) its application to email is undecided. Rule/Acceptance and the Common Law/Communicating Acceptance/Mailbox Rule is Default Rule: The mailbox rule is only a default rule, meaning that it only applies f the offeror is silent on the question of when acceptance will be effective. As always, the maker is the master of his offer, and thus the offeror is free to establish his own rules for effective acceptance. Rule/Acceptance and the Common Law/Communicating Acceptance/Mailbox Rule Dispatch of Identical Offers: In a situation where the parties simultaneously dispatch identical offers, the offers are not effective until received, and the only effect an offer has once received is to create the power of acceptance in the offeree. Accordingly, unless and until one of the parties receives and accepts one of the offers passing in the night, there is no contract. Rule/Acceptance and the Common Law/Communicating Acceptance/Mailbox Rule and Option Contracts: According to the Restatement, acceptance under an option contract is effective upon receipt by the offeror, but the case law on this question is limited. Some authorities have rejected this approach and held that the mailbox rule applies in this context as well. Rule/Rule/Acceptance and the Common Law/Communicating Acceptance/Mailbox Rule Does Not Apply: (a) Two situations under the mailbox rule involve an offeree who dispatches two responses to an offer; the first purporting to reject the offer, and the second purporting to accept it. In these cases, the mailbox rule doesn't apply. (b) The parties' obligations will depend on which of the offeree's communications reaches the offeror first, then the offeree's power of acceptance is terminated and the subsequently arriving acceptance becomes a counteroffer, which the original offeror is free to accept or reject.
Contract Formation: Offer and Acceptance 4. Acceptance and the UCC
Rule/Acceptance and the UCC: Acceptance may be made in any manner and by any medium reasonable under the circumstances [UCC 2-206(a)]. The UCC rejects the common law mirror image rule and recognizes a binding contract despite the presence of nonconforming acceptance in two situations: (1) the shipment of non conforming goods and (2) the battle of the forms. Rule/Acceptance and the UCC/Orders for Prompt or Current Shipment: According to UCC 2-206(b), an order or other offer to buy goods for prompt or current shipment shall be construed as inviting acceptance either by a prompt promise to ship or by prompt or current shipment. 1. Non-Conforming Goods Rule/Seller's Shipment of Non-conforming Goods: Under the UCC, a seller can accept a buyer's offer to purchase goods for prompt or current shipment in one of three ways: (1) a promise to ship goods in conformity with the terms of the offer, such as acknowledgement of order form sent to the buyer; (2) a prompt or current shipment of the goods in conformity with the terms of the offer; or (3) the seller can also accept the buyer's offer by shipping nonconforming goods. In contrast to common law, under which the nonconforming shipment would have constituted a counteroffer which the buyer would have been free to accept or reject, the UCC specifies that the shipment constitutes a valid acceptance and creates a binding contract between the parties. Rule/Seller's Shipment of Non-conforming Goods:EXCEPTION: Under the UCC, the shipment of nonconforming goods will not constitute acceptance if the seller notifies the buyer that the shipment is offered only as an "accommodation" to the buyer. In such circumstances, the shipment constitutes a counteroffer which the buyer is free to accept or reject. Absent accommodation, the seller's shipment constitutes acceptance of the buyer's offer under the UCC. It also constitutes a breach of the resulting contract under the perfect tender rule. 2. Battle of the Forms Rule/Battle of the Forms: (a) UCC 2-207 is designed to primarily deal with difficulties created by the application of the common law mirror image rule to commercial settings, in which parties typically transact business via the use of preprinted forms. (b) A so-called battle of the forms may occur when a buyer places an order (the offer) and the seller's acceptance form contains terms which differ from the buyer's order or are not addressed in order at all. (1) Note these rules apply regardless of which party (buyer or seller) is the offeror or the offeree. (c) Because the seller's acceptance contains terms that vary from the buyer's offer, the seller's response constitutes a nonconforming acceptance. The UCC deals with two issues in a manner different from the common law: (1) First, in many circumstances the UCC rejects the common law mirror image rule and treats a nonconforming acceptance as a legally effective acceptance, thus binding both parties to the contract despite the differing forms. (2) Second, the UCC rejects the common law last shot doctrine in determining the terms that will govern a contract formed via mismatching forms. (d) Unless acceptance is expressly made conditional on assent to the additional or different terms (a conditional acceptance), the nonconforming acceptance will operate as an effective acceptance of the offer, forming a valid and enforceable contract. Once the contract is formed the next step is to determine the effect of new or additional terms. [E.g., Buyer sends seller a purchase order for $1,000 widgets at the advertisement price of $10 each. Seller send Buyer an Acknowledgement of Order Form that promises delivery of the widgets at the stated price, but also contains boilerplate language negating warranties and limiting remedies in the event of breach. Seller's form will operate as acceptance of Buyer's offer and create a binding contract despite the presence of terms that vary from buyer's purchase order]. Rule/Battle of the Forms/Dickered versus Boilerplate Terms: Typically, the forms will be in agreement as to the dickered terms - those specific to the transaction - but may vary with respect to the boilerplate terms - standard terms that appear on the parties' respective forms regarding issues such as arbitration and warranties. When a contract is formed by an offer followed by a nonconforming acceptance, the treatment of additional or different terms depends on the identities of the parties. Rule/Battle of the Forms/Transactions Involving a Consumer: When at least one party to the transaction is not a merchant, the additional or different terms are construed as proposals to the contract. Thus, they are not part of the contract unless the offeror expressly agrees to the additional terms [2-207(2)]. Rule/Battle of the Forms/Transactions Where Both Parties are Merchants: (a) In a transaction between merchants, there is a distinction between additional terms and different terms. (1) A nonconforming acceptance contains "additional terms" when its provisions address an issue or topic not addressed in the original offer [e.g., Buyer's offer is silent on the question of arbitration, and Seller's acceptance purports to requires arbitration of all clams]; (2) a nonconforming acceptance contains "different terms" when the offer says one thing about a particular issue and the would-be acceptance says something else [e.g., B's offer requires arbitration of all claims. and seller's acceptance precludes it]. Rule/Battle of the Forms/Additional Terms: Additional terms become part of the contract UNLESS: (1) the offer expressly limits acceptance of the terms; (2) the offeror objects to the additional terms within a reasonable time after receiving notice of them; or (3) the additional terms would materially alter the contract. (a) terms that materially alter the contract are those that would result in surprise or hardship if incorporated without the express awareness of the other party; (b) examples of clauses that would materially alter the contract include: (i) a clause negating standard warranties, such as that of merchantability or fitness for a particular purpose in circumstances in which either warranty normally attaches; (ii) a clause requiring that complaints be made in a time materially shorter than a customary or reasonable; and (iii) any clause that would vary in a significant way an established usage or trade, or course of past dealing between the parties. [E.g., B sends S a purchase order for 1,000 widgets at the advertised price of $10 each. Seller sends buyer an acknowledgment of order that promises delivery of widgets at the stated price and includes boilerplate language that negates all warranties and requires payment for the order to be completed within 30 days of delivery, which is standard in the widget trade. The negation of warranties will not be considered part of the contract because it would materially alter the contract. The 30-day payment deadline would be considered part of the contract because it would not materially alter the terms of the contract.] Rule/Battle of the Forms/Different Terms/Knockout Rule: (1) With respect to different terms, the majority of decisions employ the knockout rule and omit both the offeror's original provision and the offeree's differing provision from the resulting contract; [e.g., B's purchase order contains a choice-of-law provision stating that California law will govern disputes arising from the transaction, and Seller's order acknowledgement states that New York law will govern. Neither provision will be part of the parties' contract; if the parties desire a choice-of-law provision, they will have to negotiate from scratch. (2) A minority of decisions treat a different term appearing in the acceptance as a mere proposal for alteration of contract, which the offeror is free to accept or reject; [e.g., same facts as above, the buyer's California law provision would govern the contract unless he expressly agreed to the seller's contrary provision]. Rule/Battle of the Forms/Written Confirmations: (a) Another situation governed by the battle of the forms rules occurs when parties enter into a contract-typically in real-time via face-to-face or telephonic communications-and then one or both of the parties follow up with a written confirmation containing terms additional to or different from the terms of the original deal; (b) As in the case for a contract formed by an offer followed by a nonconforming acceptance, the treatment of the additional or different terms depends on the identity of the parties: (1) If at least one of the parties to the transaction is a consumer, then the additional or different terms are mere proposals for addition to the contract which the receiving party is free to accept or reject; (2) If the transaction is between merchants: (a) any additional terms are automatically part of the contract unless: (i) they would materially alter the contract; or (ii) the receiving party objects to them within a reasonable time. (b) any terms in a confirmation that differ from the terms of the prior agreement are proposals for the inclusion in the contract, which the receiving party is free to accept or reject; and (c) if both merchants send written confirmations and those confirmations contain conflicting terms, then the knockout rule applies and neither party's term is in the contract; (i) all jurisdictions agree that the knockout rule applies to written communications; (c) because the parties already have a binding contract in this situation -formed prior to the sending of any written confirmations - a written confirmation cannot constitute a conditional acceptance and, accordingly, the parties are bound to the contract irrespective of any differences between their prior agreements and the forms. Rule/Battle of the Forms/Conditional Acceptance: Under the UCC, a definite and seasonable expression of acceptance will operate as an acceptance even though it states additional or different terms, unless acceptance is expressly made conditional on assent to the additional or different terms. HOWEVER, if the acceptance is made expressly or conditional on assent to the additional or different terms, the nonconforming acceptance will not be effective to form a contract. (a) In this situation, no contract is formed until the offeror expressly assents to the additional or different terms. Rule/Battle of the Forms/What Constitutes a Conditional Acceptance: Under the majority rule, a would-be-acceptance which tracks the language of the exceptions constitutes conditional acceptance if the language in question is clear and conspicuous; thus, the exchange of forms does not create a contract. Either party is free to back out of the deal prior to the buyer's assent to the additional or different terms. Under the minority rule, a would-be acceptance constitutes a conditional acceptance only if it clearly communicates that the offeree is unwilling to do business with the offeror unless and until the offeror agrees to the offeree's terms; under the minority rule, this exchange of forms would constitute conditional acceptance. By contrast, the proviso "our acceptance of your order is expressly conditional on your assent to the additional or differing terms that appear in this acknowledgement" would not constitute acceptance under the minority rule but would under the majority rule. A proviso reading "Upon receipt of your reply to this message stating your agreement to these terms, we will immediately ship your order" is acceptable for conditional acceptance under the minority rule. Rule/Battle of the Forms/Contracts Formed by Conduct: (a) The UCC also provides that the parties' conduct in recognizing the existence of a contract is sufficient to establish a contract, even though their writings do no otherwise establish a contract [e.g., B sends purchase order to S, S responds with a conditional acceptance containing a negation of warranties, and the parties have no further communications. Seller nevertheless ships the ordered goods, and B accepts and pays for them. Although the parties' writings do not form a contract - because S sent a conditional acceptance - a binding contract is formed by their conduct]. (b) If a contract is established in this way, the terms of the contract will be the terms on which the writings of the parties agree, together with the implied terms of the default rules established by the UCC for warranties, remedies, party performance, and other topics. [e.g., same facts, the parties' contract will conclude all the terms on which their respective forms agree - typically, the dickered terms such as the product description, price, and quantity. Seller's negation of warranties would not be included in the contract because it does not appear in the buyer's form. The UCC implied warranties would therefore govern the transaction despite Seller's attempt to negate them. Rule/Battle of the Forms/Transactions without Preprinted Forms: Although most situations governed by these rules will involve transactions consummated via preprinted forms, the same rules apply to any sale of goods in which an offer is followed by a nonconforming acceptance (e.g., an exchange of transaction-specific emails or letters) or a real-time contact is followed by a confirmation(s) (e.g., a follow-up letter confirming the details of a deal reached over lunch). Rule/Battle of the Forms/Shrink-Wrap Contracts: The authorities are divided on whether the rules of 2-207 apply to so-called shrink-wrap contracts (i.e., agreements that consumers find "shrink-wrapped" to computers and other high-tech goods once they open the boxes containing them): (1) The courts apply 2-207 typically conclude that the boilerplate terms in such agreements are mere proposals for additions to the contract, following the usual rules governing transactions that are not between merchants. (2) Other courts have held that 2-207 does not apply and that the consumer's retention and use of the goods constitute acceptance of the shrink-wrap terms.
Assignment of Rights and Delegation of Duties/Assignment of Rights
Rule/Assignment of Rights: (1) An assignment is a transfer of a right to receive a performance under a contract; (2) assignments typically have three relevant parties in a basic factual scenario: A and B have a valid contract, and B subsequently assigns his rights under the contract to C. In this scenario: (a) A is the obligor (the party with the obligation to perform); (b) B is the assignor (the party who assigned the right); and (c) C is the assignee (the party to who the right was assigned). (3) To make an effective assignment of a contract right, the owner of that right must: (a) manifest an intention: manifestation of intention is ordinarily established by the oral or written words of the assignor. (1) there are no "magic words" - not even an express reference to "assignment" - required to make a manifestation effective. (2) Oral assignments are effective unless there is some independent applicable writing requirement, such as assignment of a right to real property under a contract of sale, which must be in writing under the SOF. (b) To make a present transfer of the right without further action by the owner or the obligor. (1) A promise to transfer a currently existing right at a future date is not an effective assignment; (2) A promise to transfer a right that the assignor expect to acquire in the future is not an effective assignment [e.g., The statement "I will assign you 10% of the royalties when I obtain them" is not an assignment, as it expresses only an intent to assign in the future. (3) An "order" by an obligee directing an obligor to pay the debt to a third party is not an assignment. The most common examples of an order is a check, which under both common law and the UCC, does not operate as an assignment of the drawer's rights against the bank. Rule/Partial Assignments: A partial assignment is valid. Hence, there can be an assignment of a faction of the assignor's rights. (b) When suit is brought, however, all of the parties owning rights after the assignment must be joined in the action, unless joinder is not feasible and it is equitable to proceed without joinder. Rule/Rights to be Assigned: (a) The general rule is that all rights are assignable, subject to the following exceptions: (1) a right is not assignable if the assignment would materially alter the risks to or obligations of the other party to the contract [e.g., A and B are parties to a requirements contract under the terms of which A is obligated to supply B with B's monthly requirements for widgets. B's rights under the contract are not assignable to C if C's monthly requirements would greatly exceed B's]; (2) a right is not assignable if the obligor has a personal interest in rendering the performance in question to the obligee and not to a third party;(3) a right is not assignable if assignment would violate applicable law or public policy [e.g., a statute prohibiting the assignment of wages]; and (4) a right is not assignable if assignment is prohibited by the contract. (Note that such provisions are strictly construed in both scope and effect). (a) Although there is older precedent invalidating an assignment when the contract expressly states that attempted assignments are "void," most courts will treat an assignment in violation of contractual restriction as a breach of contract by the assignor-making him liable for any damages to the obligor-but not as a basis for nullifying the obligor's performance obligation to the assignee. (b) absent contractual language or circumstances suggesting a contrary intention of the parties, a contractual prohibition against "assignment of the contract": (i) will bar a delegation of contractual duties but not an assignment of contractual rights; will not apply to: (a) rights that accrue to the assignor as a result of a breach of contract by the obligor [e.g., If A agrees to sell and B agrees to buy widgets under a contract prohibiting assignment, and A fails to deliver the promised widgets, giving rise to a claim for breach by B, B can assign his rights upon breach to C]; or (b) rights that accrue to the assignor upon complete performance by the assignor [e.g., if A agrees to sell and B agrees to buy widgets under the terms of a contract prohibiting assignment, and A delivers the promised widgets, A can assign his rights to B's payment to C]; and (c) can be invoked by the obligor (to resist making performance to the assignee) but not only by the assignor (in an action brought by the assignee). (5) An offer cannot be assigned, although an option can be. (6) The right of a partner to share in the management of the partnership is not assignable, but the rights to profits and to the partner's share in a dissolution is assignable. Rule/Assignment for Value Contrasted with Gratuitous Assignment: (a) An assignment for value - were the assignee acquires the assignor's contractual rights in exchange for payment or a promise thereof to the assignor, is valid against the obligor and cannot be revoked by the assignor, although it may be modified like any other contract via mutual consent of the parties; (b) a gratuitous assignment - where the assignor assigns contractual rights to the assignee without consideration for such transfer, has the following legal effects: (1) a gratuitous assignment is valid and binding against the obligor, who cannot raise the absence of consideration between the assignor and the assignee as a defense to any breach of his obligations to the assignee; (2) between the assignor and the assignee, the law of gifts governs the question of whether the assignor can revoke the assignment. A gift requires that there be donative intent and delivery. (a) Because the assignment is a transfer of legal rights under a contract, it is impossible to physically deliver an intangible right. However, delivery of something representative or symbolic of an intangible right, such as a savings account book, is held to be sufficient to meet the delivery requirement. (3) Even if there is no delivery, the assignment becomes irrevocable once payment of the obligation is made to the assignee. (4) The assignor will be estopped from revoking the assignment if the assignee acts to his detriment in reliance upon the assignment. Rule/Rights and Obligations of the Parties after Assignment: (a) Rights of the Assignee against the Obligor: (1) The basic rule is that: (a) an assignee gets whatever rights to the contract his assignor had; (b) the assignee takes subject to whatever defenses the obligor could have raised against the assignor, such as a lack of consideration, incapacity, fraud, duress, or mistake. (2) Payment to Assignor - (a) if the obligor pays the assignor, this defense can be raised against the assignee, provided that payment was made before notice of the assignment was given to the obligor; (b) once notice of the assignment is given, payment to the assignor is no defense; (c) if the obligor doubts whether the assignment was made, he can pay the money into court and interplead the assignor and the assignee. (3) Setoffs and Counterclaims: (a) if the obligor has a right of setoff that could be raised against the assignor, such right can always be raised against the assignee of the alleged setoff arises out of the same transaction; (b) if however, the setoff arises out of a separate transaction, it is available against the assignee only if the transaction which gave rise to the setoff arose before notice of the assignment was given to the obligor; (c) the obligor and the assignor may agree to an adjustment of their rights without consent of the assignee up until the time that the assignee has given notice of the assignment to the obligor. (4) Waiver of Defenses - (a) if one of the original parties to the contract agrees that he will not raise defenses against an assignee in the event that the rights are assigned, the agreement is enforceable with two limitations: (1) defenses that in the nature of "real defenses" under Article 2 of the UCC can still be raised; and (a) these defenses include infancy, other incapacity that voids a contract, fraud in the execution, duress (when it removes the parties' capacity to contract), discharge in BK, and any other discharge which the assignee has reason to know. (2) The agreement not to raise defenses is invalid if the obligor who signed the waiver was the buyer or lessee of consumer goods. Rule/Rights of Assignee against Assignor: (1) Unless a contrary intention is manifested, one who assigns or purports to make an assignment for value, impliedly warrants to the assignee: (a) that he will do nothing to defeat or impair the value of the assignment and has no knowledge of any fact that would do so; (b) that the right assigned actually exists and is not subject to any limitations or defenses against the assignor other than those state or apparent at the time of the assignment; and (c) that any writing evidencing the rights that are being delivered to the assignee to induce him to accept the assignment is genuine. [note: An assignment does not in and of itself. operate as a warranty that the obligor is solvent or that the obligor will perform his obligation]. Rule/Rights Among Successive Assignees: (1) A problem of rights among successive assignees may arise when the assignor, who is owed money by the obligor, assigns his right to the money to the first assignee, and then later assigns the same right to the second assignee. The assignor is liable to both assignees for assigning the same right twice. (2) However, if the assignor is BK or has fled the jurisdiction and both assignees attempt to collect from the obligor, the majority rule is that the first assignee prevails. (a) There are various exceptions to the majority rule. In several jurisdictions, and under the Second Restatement, a subsequent assignee who has paid value and took the assignment in good faith will prevail if he: (1) obtains payment from the obligor; (2) recovers a judgment on the debt; (3) enters into a new contract with the obligor; and (4) receives delivery of a tangible token or writing from the assignor, the surrender of which is required by the obligor's contract.
Defenses 3. Duress
Rule/Duress: (1) At common law, the defense of duress was available in two circumstances: (a) physical compulsion - a party could claim duress as a defense to contract liability if the party signed the contract under force of physical compulsion; and (b) unlawful threat - a party could claim duress as a defense to contract liability if the party signed the contract under threat of unlawful activity against himself or his family. (2) Although the above two circumstance still constitute duress, the contemporary definition of the term has been relaxed to cover less egregious instances of coercive persuasion. Thus, under modern law, there are three elements to a defense of duress: (a) a threat - a threat made by the perpetrator is a manifestation of intent to inflict harm on the other person, made in words or by conduct; (b) that is wrongful in nature - (1) a threat is wrongful in nature if it involves criminal or tortious conduct, whether the threat itself is unlawful or whether it is merely a threat to engage in future unlawful action; (2) a threat to pursue criminal charges constitutes a wrongful threat, even if the perpetuator honestly and reasonably believes that the target is guilty of that crime; (3) a threat to bring civil action is wrongful only if made in bad faith - that is, if the perpetrator does not honestly and reasonably believe that the target has civil liability; (4) a threat is sufficient to constitute duress if what is threatened is a bad faith breach of the contract. This is commonly known as "economic duress" - (i) if a party refuses to perform unless he is awarded additional benefits, this may constitute a bad faith breach of contract; (ii) not every demand for a change in contract price will be considered a bad faith demand. If the demand for a change in contract terms is due to the fact that performance under the terms of the contract has become extremely burdensome because of unanticipated circumstances, there is no duress. (c) that leaves the aggrieved party with no reasonable choice but to succumb to the threat. (1) The third element of duress is that the wrongful threat must leave the aggrieved party with the absence of reasonable choice except to agree to the contract or modification on which the perpetrator is insisting. This is seldom an issue when what is threatened is a crime or a tort, a criminal prosecution, or a bad faith civil suit, for risking the eventuality of any of those is scarcely a reasonable choice; (2) hard cases often arise in the context of economic duress. Three common situations where this will arise are; (a) when there are no adequate and reasonably priced substitutes for the services or goods the perpetrator is threatening to withhold [e.g., the ship's captain is not in a position to find replacement sailors when already out to sea, and so no adequate substitutes would be available]; (b) when threatened breach would cause the aggrieved party to break his own contracts, especially if there is a prospect of substantial consequential or liquidated damages; or (c) when the alternative of acquiescing to the threat and then suing for damages is inadequate redress substantial harms to the aggrieved party [e.g., where the threatened breach would case the aggrieved party to renege on its own commitments and thus harm its reputation and opportunities for future business, damages would be inadequate as a remedy]. A minority of jurisdictions add another element to the requirements for economic duress: the aggrieved party must protest the threatened breach rather than acquiesce without complaint in demand for additional payments of benefits with the secret intention of refusing to make the newly promised payment at contract's end. The courts that impose this additional requirement hold that the obligation of good faith requires the aggrieved party to speak up at the time of the threat. Rule/Duress/Third-Party Duress: If the wrongful threat is made by a third party, rather than the other party to the contract, the aggrieved party will still have a valid claim of duress unless: (1) the other party gives value or relies materially on the transaction; and (2) the other party is proceeding in good faith without reason to know of the duress. Rule/Duress/Remedies: (a) contracts made under physical compulsion are void; (b) contracts entered into under the other forms of duress are voidable at the option of the aggrieved party; (c) the aggrieved party is: (1) entitled to restitution of any benefits conferred under duress; and (2) required to return excess value of the benefits to the perpetrator. Rule/Duress/UCC: the common law defense of duress can be used in cases arising under the UCC as a supplementary provision.
Remedies 2. Equitable Remedies
Rule/Equitable Remedies/In General: The typical remedy for contract claims is to award monetary damages. However, where an award of money damages would be inadequate to compensate the aggrieved party, a court may instead award an equitable remedy. The most common equitable remedies in contracts cases are specific performance (which requires the breaching party to take some particular action) and negative injunctions (which prohibit the breaching party from a particular action). Rule/Equitable Remedies/Specific Performance: (a) specific performance is an extraordinary remedy that is available to order a breaching party's performance only where a monetary award would be inadequate to grant relief to the aggrieved party; (b) as a practical matter, specific performance is a remedy for paying, rather than performing, parties (e.g., buyers who pay more money for goods, purchasers who pay money for real estate, and parties who pay money for services rendered). (1) when the paying party breaches, the remedy of money damages will. in the ordinary case, give the performing party the promised money, and so the availability of specific performance is a nonissue; (2) when the performing party breaches, however, the paying party may prefer to secure the promised performance itself rather than money damages; this raised the question of whether specific performance is available. (3) At common law, money damages are presumed to be inadequate when a party is purchasing either: (a) unique objects; or (b) real property. Rule/Equitable Remedies/Specific Performance/Equity Considerations: (a) specific performance is an equitable remedy, and a decision to grant or deny it is, accordingly, a decision for the court and not the jury; (b) whether to grant or deny specific performance is committed to the court's remedial discretion based on the competing equities in particular cases, and is not bound by hard-and-fast rules (such as those governing money damages); (c) the court's determination is likely to take into account some or all the following factors: (1) whether the aggrieved party has clean hands (has dealt fairly and in good faith with the breaching party); (2) whether the terms of the contract in question are sufficiently definite; (a) because a decree of specific performance is enforceable via contempt, courts are reluctant to issue an order that doesn't give the parties adequate guidance as to the conduct required. (3) whether performance by he aggrieved party can be reasonably assured; (a) for example, performance could be assured by requiring tender of the aggrieved party's performance as a condition of the breaching's party's obligations under the decree; (4) whether the terms of the contract are fair. (a) a court may deny specific performance where the underlying exchange is unfair even if the unfairness falls short of unconscionability or other doctrines that would excuse contractual performance altogether by the breaching party. (b) in some jurisdictions, a party seeking specific performance of a contact for real property must plead and prove adequate consideration, despite the general rule that court's don't police the adequacy of contractual consideration; and (5) whether specific performance would be in the public interest. (a) Specific performance may be denied, for example, where the decree might have anticompetitive results. Rule/Equitable Remedies/Specific Performance/Contracts for Which Specific Performance is Not Available: (1) Contracts for Personal Services - specific performance is not available to require a breaching employer or contractor to perform. Specific performance in such a case would violate the public policy prohibiting involuntary servitude, and it would be difficult to enforce. (2) Contracts Requiring Ongoing Cooperation Between Parties - Contracts that require a continuing series or acts or continuing cooperation between the parties for successful performance would present courts ordering specific performance with daunting problems of supervision similar to those in the personal services context; these contracts are disfavored and are not capable of immediate enforcement Rule/Equitable Remedies/Specific Performance/Sale of Goods Contracts under the UCC: (1) UCC-the UCC liberalizes the rules governing the availability of specific performance for sales of goods in two ways: (a) specific performance may be permitted if the goods are unique or in "other proper circumstances." (1) Thus, the subject matter of a contract does not have to be unique in order to obtain specific performance; if the buyer has adequately searched but is unable to cover the breach, "other proper circumstances may be present; (b) specific performance is available even where ongoing cooperation would be required between the parties (e.g., output requirements contracts) so long as the requisite inability of a party to cover can be established. Rule/Equitable Remedies/Specific Performance/UCC/Replevin: The buyer also has a right of replevin - which is an action to repossess property-for goods identified in the contract if: (1) after reasonable effort, the buyer is unable to cover; (2) the circumstances reasonably indicate that an effort to cover will be unavailing; or (3) if the goods have been shipped under reservation (i.e., the seller has reserved a security interest in the goods) and satisfaction of the security interest in them has been made or tendered. Rule/Equitable Remedies/Negative Injunctions: (a) Negative injunctions are orders by the court prohibiting the breaching party from taking a particular action. The limitations that apply to specific performance also apply to negative injunctions. Most commonly, negative injunctions are used to prevent employees from going to work for a competitor or competing with their former employer. (b) The availability depends on whether the former employer is seeking mid-term or post-term relief: (1) midterm relief - (a) when an employee is under a contract for a specified period of time and the employee breached the contract by departing before the end of that period, a negative injunction will be available to prevent the employee from competing directly or indirectly with his former employer if the employee's services are unique or extraordinary; (b) most of the cases finding that services meet this test involve professional athletes and entertainers. (1) Although the presence of a specific contractual provision establishing exclusive employment during the period of time in question will aid the employer in securing a negative injunction, most courts imply such a term for the period of employment specified in the contract [e.g., an opera singer under a contract to sing at Her Majesty's Theatre for a three-month period was persuaded to depart mid-contract to begin a concert series at a competing venue. Because her services were unique and extraordinary, the court granted a negative injunction barring her from performing at any competing venue for the duration of the contract term, Lumley v. Wagner]. (2) Post Employment Relief: (a) Enforcement of employment contract provisions that prohibit post-employment competition against the employer have become a common source of negative injunctions. (b) The validity of such noncompetition clauses will depend on the following three factors: (1) most courts require a significant business justification for enforcing post-employment restraints; (a) An employer's need to protect confidential information would qualify, but some courts will allow little or no justification for enforcing such a provision; (2) most jurisdictions limit the scope of the enforcement of non- competes to a reasonable duration and geographical restrictions; (a) HOWEVER, the advent of the information age has prompted courts to broaden the definition of a reasonable geographical restraint, but at the same time, to lessen the amount of time considered to be a reasonable duration of restraint; and (3) most courts will issue a negative injunction against a competing employer only if the employment contract contains an express noncompetition agreement, but some courts have granted relief in the absence of such a provision based on the tort theory of inevitable disclosure of trade secrets.
Performance, Modification, and Excuse 3. Excusing Performance Due to Faulty Assumptions A. Mistake B. Impossibility C. Impracticability D. Frustration of Purpose
Rule/Generally: A party entering a contract is susceptible to making countless assumptions about the present and the future circumstances in which performance will take place. When such assumptions turn out to be faulty, the doctrine of mistake, impossibility, impracticability, and frustration of purpose may be available to excuse the parties' obligations [e.g., (a) A faulty assumption about the facts at the time of contracting would be that stone in a ring the seller is selling is a diamond and not a topaz; (b) a faulty assumption about future facts in a contract would be the assumption that the cost of fuel won't increase beyond a certain range and triple the costs of the busing service whose bid was accepted by the school district]; (2) faulty assumptions about present facts, or the facts as they exist at the time of contracting, are dealt under the doctrine of mistake. Faulty assumptions about future facts are dealt via the doctrine of impossibility, impracticability, and frustration of purpose. A. Mistake Rule/Mistake: (a) When a party or parties make a faulty assumption about the present circumstances, and thus enter a contract on that basis, this is known as a mistake. (b) A mistake regarding the facts that exist at the time contracting will excuse performance only where the mistaken facts are material to that contract. This requires that the mistaken facts will significantly impact the value of the transaction to one or both parties. (c) When only one of the parties to a contract is operating under a faulty assumption about material facts as they exist at the time of contracting, the situation is governed by the rules of unilateral mistake. When both parties labor under a common faulty assumption, the situation is governed by a mutual mistake. Rule/Unilateral Mistake: (a) Under the rules governing mutual mistake, a party operating under a faulty assumption about material facts as they exist at the time of contracting is not excused from his contractual performance unless: (1) the other party knew or had reason to know of the mistake; or (2) the mistake was based on a clerical error; (a) the clerical error exception is not available in all jurisdictions, and where available, it is subject to the following exceptions: (i) the fact about which the parties were mistaken is essential to the contract (i.e., it goes to the very heart of the exchange; (ii) both parties were mistaken; and (iii) the disadvantaged party did not bear the risk of mistake under the parties' agreement [e.g., (1) A agrees to sell B a cow at beef prices because at the time of contracting, both parties were under the assumption that the cow was barren. A short time later, the cow was discovered to be with calf, which greatly increased her resale value. The contract is voidable at the option of A; (2) same facts, but B tells A that he is still going to try and breed the cow in any case. In this case, A bears the risk of B's efforts succeeding, because if B succeeds, it will prove the parties' assumption that the cow was barren wrong. So A is accordingly bound by the contract even if the cow turns out to be fertile]. Rule/Mistake/Application under the Common Law and the UCC: (a) The doctrine of mistake originated under common law, but is also available in sales of goods cases governed by the UCC. Impossibility, Impracticability, and Frustration of Purpose: Rule/Generally: (a) where the faulty assumptions of the parties relate to future facts, as opposed to existing facts, these issues are dealt with under the doctrine of impossibility, impracticability, and frustration of purpose; (b) These doctrines may used as a defense to a breach of contract claim. HOWEVER, they can also be raised affirmatively for claims of rescission or cancellation of the contract. B. Impossibility Rule/Doctrine of Impossibility: (1) The doctrine of impossibility excuses both parties from their obligations under a contract if the performance has been rendered impossible by events occurring after the contract was formed. (2) Application of the doctrine of impossibility requires: (a) objectively impossible performance - occurs when the performance under the contract becomes literally impossible because of circumstances beyond control of the parties [e.g., If X promises to sell Y his horse, but the horse dies before delivery, the X's performance has become objectively impossible]; and (b) subjective impossibility - occurs when the performance under the contract becomes impossible because of some failure or fault on the part of the performing party. Under those circumstances, the performance obligation is not excused and will be conserved as a breach of the contract [e.g., the failure of a party to have enough money either to make a promised payment or obtain the components required for the production of a promised product would be subjective impossibilities] (Note: A supplier's, of another's goods/products, failure to secure a backup plan will not meet the legal test for impossibility). (b) The occurrence of the contingency must not be shown to the parties at the time of contracting. This can occur in two ways: (1) it may be a supervening contingency, whereby the performance was possible at the time of contracting, but afterward a contingency occurs that renders the performance impossible; or (2) it may be an existing contingency, whereby a contingency existed at the time of contracting, but was unknown to the parties until after the contract was formed. Rule/Doctrine of Impossibility/EXCEPTIONS: (a) the doctrine of impossibility will not apply where the parties have allocated the risk of the contingency and provided remedial measures in he event of its occurrence; (b) if the events render performance only temporarily impossible, then this will typically only suspend obligations of the parties until the impossibility ends. Rule/Doctrine of Impossibility/Types of Impossibility: (a) There are three main categories of impossibility: (1) Destruction of the subject matter of the contract (a) where the contract performance involves particular goods, a building structure, or any other tangible item, the destruction of which occurs without the fault of either party, then the contract is discharged [e.g., B promises to buy F's 2008 wheat crop, the entirety of which is destroyed just before harvest by wheat blight. F's performance is excused on the basis of impossibility) ; (2) death or incapacity (a) if the existence of a particular person is necessary for the performance of a contract (e.g., personal performance by an individual is required, or performance is to be rendered to a specific person) then that person's death or incapacity will trigger the doctrine of impossibility, and the parties' obligations will be dismissed); (3) illegality (a) the doctrine of impossibility will discharge the parties' obligations if performance is prohibited by a change in a constitution, statute, administrative regulation, or municipal ordinance, or by judicial order. C. Impracticability Rule/Doctrine of Impracticability: (1) Courts are reluctant to excuse performance for any reason other than impossibility. HOWEVER, under the doctrine of impracticability, a promisor may be excused from performance where unforeseen difficulties have made performance prohibitively expensive or otherwise burdensome; (2) the doctrine of impracticability has been adopted into common law of contracts as well; (3) the following elements are required to show that performance under a contract would be impracticable: (a) the impracticability of the performance was caused by some unforeseen contingency; (b) the risk was neither assumed nor allocated by the parties; and (c) the increase in the cost of performance would be far beyond what either party anticipated: (1) Even the courts have allowed relief due to impracticability have held that increased costs alone is not sufficient as an excuse. Rule/Doctrine of Impracticability/Under the UCC: (a) Under the UCC, a number of examples of contingencies that would not excuse performance are listed: (1) increased costs alone does not excuse performance unless the rise in cost is due to an unforeseeable contingency and alter the essential nature of performance; (2) A rise or collapse in the market will not justify impracticability because those contingencies are exactly the types of business risks that fixed-price contracts are intended for. (b) The UCC also lists the following examples of contingencies that would trigger impracticability. All involve as severe shortage of raw materials or supplies: (1) where the shortage is caused by: (a) war or embargo; (b) local crop failure; or (c) unforeseen shutdown of major sources of supply; or (2) the shortage either caused a marked increase in cost or prevents the seller from securing the supplies necessary for his performance. Rule/Doctrine of Impracticability/Contractual Allocation of the Risk of Non-Performance: (a) The parties are free to allocate the risk of a contingency or to specify the remedial measure to dictate performance in the event of a contingency. Where they do so, the performance that would otherwise be excused by impracticability will be governed by their contractual agreement; (b) the doctrine of impracticability thus operates as a default rule where the contract is silent as to the occurrence of a contingency; (c) whether the parties have provided for the contingency will depend on all of the circumstances, but some specific guidelines apply. (1) If the contingency was foreseeable at the time of the contract and the contract is silent, it may suggest that the absence of a term excusing the performance was an affirmative decision to allocate the risk to the performing party; (2) the payment of a supra-market premium by the non-performing party might also reflect that the parties bargained for the contingency, and that the consideration in exchange for the premium would be the performance of the party in spite of the contingency; (3) where evidence of a course of dealing or of trade usage reveals that it is customary for the performing party or others in a similar position to assume the risk of such contingencies. Rule/Doctrine of Impracticability/Foreign Domestic Relation Order: A performance obligation may be excused when the failure to perform is based on a good faith compliance with a foreign or domestic regulation or order. D. Frustration of Purpose Rule/Doctrine of Frustration of Purpose: (1) Where a contingency occurs that dramatically reduces the value of performance to the receiving party, the doctrine of frustration of purpose may be available to excuse the receiving party from its contractual obligations [e.g., the owner of a London flat with a "ringside" view of the forthcoming coronation parade who agreed to lease the flat at premium prices to a lessee eager to witness the festivities. The parade was canceled when the king became ill, and the lessee's contractual obligations to the owner were excused on the grounds that going through with the rental agreement in the absence of its raison d'etre cannot reasonably be said to have been in contemplation of the parties at the date of the contract. Krell v. Henry]. (2) Modern Test: (a) The contemporary version of this rule will discharge a party's contractual obligations when the following three conditions are met: (a) the party's principal purpose in entering the contract is frustrated - this means that the frustration of incidental or nonmaterial purposes would not qualify; (b) there is substantial frustration - thus, a variation on Krell v. Henry in which the town erected bleachers that only partially obstructed the parade view from the apartment would not justify a finding of frustration; and (c) non-occurrence of the event precipitating frustration was a basic assumption of the contract. Rule/Doctrine of Frustration of Purpose/Contractual Allocation of the Risk: (a) Where the parties contractually allocate the risk of non-occurrence of the event, the doctrine of frustration will not apply. (b) this is also applicable where the risk are foreseeable [e.g., same facts in Krell but lessee heard the buzz around town of the king's possible serious illness days before entering into the contract]. Rule/Doctrine of Frustration of Purpose/Application under Common Law and the UCC: (a) the frustration of purpose doctrine is followed under the common law; (b) the frustration of purpose doctrine is available to supplement the UCC in sales of goods contracts.
Substitute Performance/failure/unavailability -UCC 2-614:
Where without fault of either party, the agreed...facilities fail or a particular carrier becomes available or the agreed manner of delivery becomes commercially impracticable but a commercially reasonable substitute is available, such substitute must be tendered an accepted. E.g., Golfer Corp., pays Tiger Woods to promote new golfing line at a White House Dinner; on the evening of the dinner Woods fell ill and was unable to promote the new line; he may escape liability by offering an equivalent substitute performance.
Sources of Contract Law 1. In General
1. There a two principal sources of contract law: Article 2 of the UCC and state common law. 2. In deciding which source of contract law applies, first look for the subject matter covered by the UCC, if the UCC does not apply, then common law will apply.
Third Party Beneficiaries 1. In General 2. Classification of Third-Party Beneficiary 3. Rights of Parties to Enforce the Contract MBE: Quick Rule Summary Rule/Incidental Beneficiaries: With respect to third party beneficiary contracts, intention to confer a benefit on the third party is essential to the right of third party to enforce the promise. If performance to be rendered to the third party, she is a protected beneficiary, and thus entitled to sue. But if the promised performance is to be rendered to the promisee, the contract is for the sole benefit of the parties thereto, and any third party is an incidental beneficiary. Rule: Even though the beneficiary is not in direct privity with the promisor; they can still sue as an intended beneficiary.
1. In General Rule: In a typical contract, the parties promise performances to each other. If one party refuses to perform, the other party has standing to bring a claim for breach because the parties are in contractual privity. In some contracts, however, one of the parties promises a performance that will benefit a third party (a third-party beneficiary). The critical issue is third-party beneficiary law is the circumstances under which the third-party beneficiary has standing to enforce the contract against the promisor. (a) Under common law, courts traditionally held that a third-party beneficiary did not have standing to sue the promisor because there was no contractual privity between them: however, under contemporary law, the privity requirements has been relaxed and third-party beneficiaries may have standing to sue the promisor for breach, even though the promise was made to the promisee and not the third-party beneficiary. 2. Classification of the Third-Party Beneficiary Rule/Generally: The right of an aggrieved third-party beneficiary to bring an action for breach of contract against a breaching promisor or promisee will depend upon the classification of the beneficiary. The First and Second Restatement each classifies beneficiaries differently. (a) While the two Restatements use different terminology, the practical effect is the same under both nomenclatures. Rule/First-Restatement/Third Party Beneficiaries: (a) The First Restatement placed-third party beneficiaries into one of three categories: (1) creditor beneficiary-when a promisee seeks a performance from the promisor that will benefit a third party, and the promisee's purpose is to satisfy a debt or other obligation owed by the promisee to the third party, the third party beneficiary is the creditor beneficiary [e.g., A owes B $100 and enters a contact with C to whitewash C's fence in exchange for C's promise to pay B the $100. B is a creditor beneficiary because A's purpose in contracting with C is to satisfy his debt to B]; (2) donee beneficiary-when a promisee seeks a performance from the promisor that will benefit a third party, and the promisee's purpose is to make a gift of that performance to the third party, the third party beneficiary is a donee beneficiary [e.g., Desiring to make a graduation gift to A, B agrees to pay C $1000 in exchange for C's promise to whitewash A's fence. A is a donee beneficiary because B's purpose is contracting with C is to make a graduation gift to A]; and (3) incidental beneficiary-third parties who will benefit form a promisor's performance as a practical matter, but who may not meet the test for creditor or donee beneficiaries under the first restatement are incidental beneficiaries. Rule/Second Restatement/Third-Party Beneficiaries: The Second Restatement eliminates the "creditor" and "donee" beneficiary terminology, and instead puts third-party beneficiaries into one of two possible categories: intended beneficiaries and incidental beneficiaries. (b) Intended beneficiaries fall into the same two categories utilized by the First Restatement, where the promised performance will satisfy an obligation to pay money to the beneficiary (creditor beneficiaries) and where the promisee intends to give the beneficiary the benefit of the promised performance (donee beneficiaries). Many courts continue to use the older terms however. (1) The key factor in determining whether party's is an intended beneficiary is whether the promised performance is intended to benefit the third party-beneficiary. Courts are divided over whether the test for determining intent to benefit requires a mutual intention of the parties (the Second Restatement Test) or merely a reasonably apparent purpose of the promisee (the First Restatement Test). [e.g., A hires Attorney B to draft A's will in accordance with A's instructions, which include a wish to make A's stepdaughter, C a residual heir. Attorney B breaches the contract by neglecting to include the requested provision, and upon A's death, C brings a third-party beneficiary action against Attorney B for his breach. The intention of A to benefit C was "reasonably apparent" to Attorney B and, accordingly, would satisfy the "intent to benefit" test under the First Restatement. But because the benefit to C was a matter of indifference to Attorney B, there would be no "intent to benefit" her under the Second Restatement "intention of the parties" test. (c) Incidental beneficiaries-as under the First Restatement, are third parties who will benefit form a promisor's performance as a practical matter, but who may not meet the test for intended beneficiaries under the Second Restatement. Rule/Third-Party Beneficiaries/Rights of Parties to Enforce the Contract: (a) Under both the First and Second Restatement, an incidental beneficiary does not enjoy any right to seek enforcement of the contract from either promisor or promisee to the original agreement. (b) Any third-party beneficiary, other than the incidental beneficiary, has a right to secure enforcement of the agreement from a breaching promisor. (1) This applies to creditor and donee beneficiaries under the First Restatement and intended beneficiaries under the Second Restatement. Rule/Third-Party Beneficiaries-Third-Party's Rights against the Promisee: (1) Although a third-party beneficiary has rights against the promisor in connection with the promised performance, he has no rights against the promisee in the event that the performance is not forthcoming [e.g., Desiring to make a graduation gift to A, B agrees to pay C $100 in exchange for C's promise to whitewash A's fence. If C fails to whitewash A's fence, A has no recourse against B]. (2) Instead, a third-party beneficiary will only have rights against the promisee resulting from the promisor's failure to perform based on whether there is an independent obligation between the promisee and the third-party beneficiary. (a) Thus, a First Restatement creditor beneficiary can elect to sue the promisee on the prior obligation between the beneficiary and the promisee or the promisor on the third-party beneficiary contract. However, satisfaction of the claim by one operate as a release of the other's obligation. (b) Similarly, a Second Restatement intended beneficiary can only sue the promisee if the beneficiary would have fallen into the First Restatement's definition of a creditor beneficiary. That is, the intended beneficiary can only sue the promisor on the third-party beneficiary contract or the promises if the promisee owes a separate obligation to the beneficiary, which the third-party beneficiary contract was intended to satisfy; [e.g., A owes B $100 and enters into a contract with C to whitewash C's fence in exchange for C's promise to pay B the $100. In the event of nonpayment, B can elect to sue either A on the pre-existing obligation or C as a third-party beneficiary, but payment of the $100 by either A or C will release the other from any other further performance obligation to B]. (3) The promisee's preexisting obligations to a creditor or intended beneficiary are not discharged by the formation of a third-party beneficiary contract under which the promisor promises to fulfill to the promisee's obligation to the beneficiary [e.g., A owes B $100 and enters into a contract with C to whitewash C's fence in exchange for C's promise to pay B the $100. C's obligation to pay B the $100 does not discharge A's preexisting obligation to B. Rule/Third Party Beneficiaries/Vesting of the Right to Sue: (1) A third-party beneficiary does not automatically have the right to seek enforcement of the contract, or prevent the contract from being modified by the original parties, purely by virtue of the third-party beneficiary contract been made; (2) the parties to a contract are free to modify or rescind it by mutual consent, and they may modify or rescind a third-party beneficiary provision without the beneficiary's consent unless and until the beneficiary's rights under the contract have vested. (a) Once the third-party beneficiary's rights vest, the ability of the original parties to make any future rescissions or modifications will be terminated; (b) if the third-party beneficiary's rights under the contract vest before he is notified of any modification or rescission of the contract, the modification or rescission will be ineffective. (3) Vesting occurs with regard to an intended beneficiary when: (a) the beneficiary brings suit on the matter [e.g., A contracts with B to pay B $2,000 in exchange for B doing renovations to C's house. B begins to perform, but ceases midway and informs C that he does not intend to complete the work. A and B agree to modify the original contract to instead have B do the renovation on A's house, thereby eliminating C from the arrangement. Before being notified of the modification, C sues B to enforce the original contract based on B's anticipatory repudiation. C's filing suit vests his rights in the agreement, and A and B's modification will be ineffective]; (b) the beneficiary changes his position in justifiable reliance on the contractual promise; (c) the beneficiary manifests his assent to the contract at the request of either the promisee of the promisor-once assent is given the parties can no longer modify the beneficiary's rights to sue; or (d) the rights of the beneficiary have vested under an express term of the contract providing for such vesting. Rule/Third Party Beneficiaries/Defenses Available to the Promisor: (1) Because a third-party beneficiary's rights are entirely dependent on the underlying contract, any valid defenses the reneging promisor would have to the enforcement of the contract, such as impracticability, the failure of a condition, or a breach of the contract by the promisee, would also be effective against the beneficiary; (2) the promisor may not assert defenses based transactions with the promisee, such as the right to setoff arising in connection with a deal unrelated to the contract at issue, unless the contract expressly subjects the rights of the beneficiary to such a claim. Rule/Third Party Beneficiaries/Promisee's Rights against the Promisor: (a) when the promisor does not perform, the promisee has a claim for breach of contract against the promisor. (b) if the promisor's performance is intended to benefit a donee beneficiary, the promisee ordinarily will not have suffered any economic loss for the non-performance and, therefore, may not be able to recover more than nominal damages. As a result, however, some courts consider the damage remedy inadequate and will order specific performance [e.g., Desiring to make a donation to her alma mater, Laurie Lawyer promised to provide legal services to Nellie Merchant, a seller of computer hardware, in exchange for Merchant's promise to provide 100 free laptops to Nutmeg Law School. If merchant fails to deliver the promised laptops, Lawyer can sue her for breach of contract, but because Lawyer has suffered no economic loss on account of the breach, she may be able to secure specific performance]; (c) If the promisor's performance is intended to benefit a creditor beneficiary, the promisee may secure specific performance on the promisor's obligation. (1) A claim for money damages would, however, expose the promisor to the possibility of double liability, since the is also liable to third-party beneficiary. Accordingly, some courts refuse to allow the promisee to recover damages against the promisor unless the promisee has already made payment to the beneficiary to cover the default.
Remedies/Monetary Damages under the UCC A. Seller's Remedies [MBE Note] - UCC 2-703 - provides where a buyer wrongfully rejects or revokes acceptance of goods on or before delivery ...then with respect to the whole undelivered balance, the aggrieved seller may (a) withhold delivery of such goods; (b) stop delivery by any bailee; (c) resell and recover damages; (d) recover damages for non-acceptance; or (e) sell the goods either by public or private sell. B. Buyer's Remedies [MBE Note] - Right to Cancel: Where the time set performance has passed, the party awaiting performance may agree to a new commercially reasonable time for performance prior to cancellation.
2. Monetary Damages under the UCC A. Seller's Remedies Rule: After there has been a rightful rejection of the goods, the buyer has a duty to: (a) follow reasonable instructions received from the seller with respect to the goods and, in the absence of such instructions, (b) make reasonable efforts to sell them for the seller's account. When the buyer sells the goods, he is entitled to reimbursement out of the proceeds for reasonable expenses of caring for and selling them. The amount however should not exceed 10 percent on the gross proceeds [UCC 2-603]. Rule: Upon a buyer's breach of a contract for the sale of goods, the seller is free to cancel the contract and to withhold delivery of any yet to be delivered goods. In addition, the seller may have a right to recover money damages for the breaching buyer. Rule: If some or all of the goods have not been delivered - either because the buyer has reject them, or in the context of anticipatory repudiation - the seller can recover the damages with respect to them. Rule/Seller Resells: If the seller resells, he can recover the contract-resale differential - that is, the difference between the contract price and the resale price [UCC 2-706]. In order for the seller to recover, the resale must be made in good faith and in a commercially reasonable manner (a seller may not make a "sweetheart" deal to sell the goods at below-market discount to a friend or relative); the seller is not accountable to the buyer for any profit made on any resale [UCC 2-708]. Rule/Seller Does Not Resell: If the seller does not resell, he can recover the contract-market differential - that is, the difference between he contract price and the market value of the goods at the time an place of the promised delivery [UCC 2-708]. Rule/Whether or Not the Seller Resells: Whether or not the seller resells he is entitled to incidental damages - that is, the costs associated with getting stuck the goods the seller though he had sold as well as the costs of resale. HOWEVER, the seller's damages will be reduced by an amount reflecting expenses avoided on account of the breach, Lost Profits for Lost Volume Sellers/Rule: A lost volume seller is one whose supply of goods exceeds he demand for the same; that is, the seller can satisfy all potential buyers who may seek to deal with him. The UCC accordingly permits lost volume sellers to recover the profit they would have made on the lost sale rather than relegating them to either the contract-market or contract-resale differential. To recover lost profits, the seller must be able to show: (1) that he could have made the sale of both the breaching and resale buyer; (2) that it would have been profitable for the seller to make both sales; and (3) that he probably would have made the additional sale to the resale buyer even absent the buyer's breach. B. Buyer's Remedies Rule: Upon a seller's breach of contract for the sale of goods, a buyer can either recover damages or seek specific performance (specific performance only if the goods are unique). There are two ways to measure a buyer's damages under UCC, the key factor is whether the buyer covered (purchased replacement goods). Rule/If Buyer Covers: If the buyer covers, his damage measure is the contract cover-differential - that is, the difference between what the buyer would have paid under the contract and what he actually paid to secure cover (replacement goods). Cover must be made in good faith and without unreasonable delay [2-712(1) and (2)]. Rule/If Buyer Does Not Cover: The measure of damages for non-delivery or repudiation by the seller is the difference between the market price at the time when the buyer learned of the breach and the contract price together with incidental and consequential damages [UCC 2-713]. Rule/Whether or Not Buyer Covers: Whether or not the buyer covers, he is also free to seek incidental damages - the costs associated with securing cover - and consequential damages. HOWEVER, the buyer's damages will be reduced by an amount reflecting expenses avoided because of the breach. Rule/Difference in Value Damages: This measure of damages available under the UCC is similar to common law expectation damages. It is available if the buyer receives nonconforming goods from the seller. The difference in value damages does not require a showing of foreseeability - Only that the goods did not conform to the goods specified in the contract. Formula: Value of the goods contracted for - Value of the goods received = Buyer's recovery under Difference in value damages Rule/Deduction of Damages from Price Still Due: A buyer may deduct all or any part of the damages resulting from any part of the price still due under the same contract. In order to make the deduction, the buyer must give notice to the seller of his intention to withhold all or part of the price [UCC 2-217, cmt 2].
SOF 6. Effect of Satisfying SOF 7. No mutuality of Obligation under the SOF 8. Limitations on the SOF
6. Effect of Satisfying SOF Rule/Effect of Satisfying SOF: (1) An otherwise perfectly valid contract, possessing offer, acceptance, and consideration, can still be unenforceable under the SOF. This can happen if the contract is covered by the SOF, but does not satisfy the SOF; (2) on the other hand, satisfaction of the SOF does not ensure a "win"; the aggrieved party must still go on to establish both a valid contract and breach thereof to succeed on his claim; (3) thus, satisfying the SOF is necessary but not sufficient alone to enforce the contract [e.g., A sues B for breach of an oral contract for the sale of land, and B raises the SOF defense. A adduces the written offer signed by B and thus establishes that the SOF is satisfied. A may still lose his breach of contract claim, however, if it turns out that the offer lapsed or was revoked before A accepted it or for any other reason A fails to persuade the trier of fact that a valid contract existed between the parties. 7. No Mutuality of Obligation under the SOF Rule/No Mutuality of Obligation under the SOF: It is possible for one party to prevail under a SOF defense while, for the same transaction, the other party would be fully liable under the contract. This is a result of the fact that the SOF does not require that both parties have a signed writing, only that the party against whom enforcement of the contract is sought has signed. If the writing evidencing the contract is signed by only one of the parties, the contract will be enforceable against the signing party but not against the non-signing party [e.g., A and B enter an oral contract for the sale of land and the only signed writing between them is an offer signed by B. If B breaches, he will not have a SOF defense because A will be able to adduce a writing signed by B. If A breaches, however, he will be able to raise a successful SOF defense because he is the party against whom enforcement is sought and he didn't sign the writing] (a) An EXCEPTION to this principle is created by the merchant's confirmation provision of UCC, under which a signed written confirmation sent by one party to the other will defeat the SOF claim by the receiving party as well as by the sending party unless certain conditions are met. 8. Limitations on the SOF Rule/Limitations on the SOF: The only effect of a successful SOF defense is to defeat enforcement of the contract against the non-signing party. The contract may still be valid and enforceable for other purposes, such as: (a) the contract may provide evidence in establishing an element of a legal claim apart from breach of contract [e.g., A brings an action against C for tortious interference with A's oral land contract with B. Although B would have a successful SOF defense in a breach of contract brought by A, the SOF will be irrelevant in A's tortious interference claim against C]; (b) the contract may provide evidence establishing a defense to a legal claim apart from breach [e.g., Under an oral contract for the sale of A's land to B, B takes possession. A reneges and brings a trespass action against B. Although A would have a successful SOF defense in a breach of contract action brought by B, the SOF would be irrelevant to the validity of B's defense to A's trespass claims]; (c) the contract may provide evidence of the value of services already rendered [e.g., A spends three month working for B under an oral two-year employment contract. If B fires A and successfully raises a SOF defense against enforcement of the oral contract, A may nonetheless be able to recover from B the reasonable value of the services rendered in quantum meruit. Moreover, he may offer the price agreed to in the oral contract as evidence of the value of the services rendered.
Performance, Modification, and Excuse 5. Conditions a. Conditions b. Promissory vs. Pure Conditions c. Express vs. Implied Conditions d. Rules Governing the Order of Performance
A. Conditions Rule/General: In some contracts, the obligation to perform is conditioned upon the happening of some event or upon action by the other party. The law regarding conditions govern two distinct issues: the order of parties' performances and the remedies where conditions to the contract have failed. B. Promissory vs. Pure Conditions Rule/Promissory versus Pure Conditions: A condition to contractual performance can be a condition controlled by the parties or can be a condition outside of either party's control. (1) A promissory condition is where the contract performance is conditioned on the occurrence of the promised performance by the other party [e.g., a public contract contains the following language: "Publisher's duty of payment is expressly conditioned on Author's timely completion of the promised manuscript." In this case, Author's timely delivery of the manuscript is a promissory condition on Publisher's contractual obligation of payment]. (2) Pure conditions are typically where contract performance is conditioned on the occurrence of events beyond the control of either party [e.g., Skyblasters, Inc. agrees by contract to provide a fireworks display for a holiday celebration sponsored by the Village of Remulak, and the contract contains a "weather permitting" provision. In this case, good weather is a pure condition on Skyblaster's performance obligation. C. Express vs. Implied Conditions Rule/Express vs. Implied Conditions: (a) Express conditions are those which the parties expressly include in provisions of the contract. (b) Implied Conditions are those created under common law or the UCC to address order of performance and rights upon breach when the parties haven't done so expressly. Implied conditions are those fairly to be inferred from evidence of the parties' intentions. They are often referred to as implied-in-fact, since their existence is determined by the process of contract interpretation rather than the express language of the contract. D. Rules Governing the Order of Performance Rule/Rules Governing the Order of Performance: The rules regarding the order of performance are dependent on whether the contract falls under the common law or the UCC. Rule/Cases Governed by the Common Law: (a) if the contract contains express conditions specifying the order of performance, then those specifications will control the order of performance; (b) if the contract is silent as to the order of performance: (1) Where one party's performance requires a period of time to be completed and the other party's performance does not, then the performance over time is treated as an implied condition of the latter. (2) Where the parties can exchange performance more or less simultaneously, then the performances are treated as concurrent conditions of each other. Rule/Sales of Goods under the UCC: Under the UCC, the parties are free to specify the order of performance, and where the contract contains those specifications, the performance obligations under that contract will be performed according to the contractual terms; (b) because sales contracts most commonly involve delivery and payment of goods, performance is treated as concurrent, and so each performance is conditioned on the performance of the other.
Sources of Contract Law 4. Basic Definitions and Concepts of Contract Law: Categories of Contractual Obligations: A. Express Contractual Obligations B. Implied-in-fact Contractual Obligations C. Implied-in-law Contractual Obligations D. Quantum Meruit
A. Express: Where the parties make oral or written expressions of their commitments; (e.g., email or face to face oral agreement). B. Implied-in-fact: Are consensual agreements that fail to express the agreement of the parties in it entirety; (e.g., an obligation to pay the "reasonable value" of services rendered or goods delivered where prices is not discussed by parties in advance. C. Implied-in-law Contractual Obligations (quasi-contract: by court order): arises where there is an equitable imposition of a would be contract. It is an equitable remedy available to prevent unjust enrichment, and arises where one party bestows a benefits on the other. An implied-in-law obligation is also commonly referred to as a quasi-contractual obligation or a restitutionary obligation. In many cases the court will conclude that the bestowal of the benefits unjustly enriched the recipient and impose and implied-in-law obligation on the part of the recipient to restore the value of the benefits conferred to the other party. E.g., emergency services to a patient, contractual benefits conferred by a mistake (merchant mistakenly delivers goods to the wrong place), or where contractual benefits have been conferred via an unenforceable contract, (buyer makes improvements on property that has not been satisfied by the SOF). D. Quantum Meruit: is a means of enforcing both implied-in-law and implied-in-fact contractual obligations. A reasonable sum of money to be paid for services rendered when stipulated contract is in effect. E.g., plumber fixes tenants home without first discussing the price; the plumber may enforce the homeowners implied-in-fact obligation to pay the reasonable price of the repair via a quantum meruit action. E.g., supplier mistakenly delivers goods to shopkeeper intended for another buyer, the supplier may seek to enforce the shopkeeper's implied-in-law obligation to pay a reasonable value of the goods via a quantum meruit action. Rule/Signed Writing Requirements: A number of rules in contract law require a "signed writing: in order to create an enforceable legal obligation [e.g., the SOF under both common law and the UCC bars enforcement of certain contracts unless the party against whom enforcement is sought has signed a writing evidencing the underlying agreement]. [e.g., The UCC requires that firm offers must be made in writing and signed by the offeror in order to be enforceable]. (2) Electronic signatures are legally effective in the vas majority of the U.S. jurisdictions under the terms of the Uniform Electronic Transactions Act. (3) The majority of courts have held that e-mail and the like are sufficient to satisfy the writing requirements of contract law.
Sources of Contract Law 3. The Common Law of Contracts
Rule/Common Law of Contracts: Contracts that do not involve the sale of goods are covered by the common law of contracts. Frequently encountered examples are service contracts, contracts involving real property, and assignments of legal claims. Rule/Interplay Between Sources/Common Law Filling in the Gaps of the UCC: (1) In cases involving the sale of goods, the UCC supplies the primary law and displaces contrary common law rules. HOWEVER, where the common law rules are not displaced, those rules will supplement the UCC and fill in its "gaps." [E.g., An example of a UCC provision displacing a common law rule would be 2-205, the firm offer rule. The firm offer rule eliminates the common law requirement for consideration by allowing a merchant to make a firm offer that will remain open by requiring a signed writing to that effect.] [E.g., Another example of UCC provisions displacing the common law rules would be 2-207, the battle of the forms provision which overrules the common law mirror image rule, and 2-209, the rule governing midterm modifications which eliminates the common law pre-existing duty rule.] [E.g., some examples of common law gap-filling include common law doctrines of fraud, duress, and incapacity of minors and/or the mentally infirm. These common law rules apply even where the sale is otherwise governed by the UCC.] Rule/UCC by Analogy: For cases that do not involve the sale of goods, the UCC does not apply. HOWEVER, courts will frequently use its provisions "by analogy" in developing common law.
Performance, Modification, and Excuse 2. Modification
Rule/Generally: At common law, parties were free to enter a contract on virtually any terms that they wished. Once they had entered the contract, however, the law made it very difficult to modify those terms. The contemporary rules allowing for modification under the UCC and common law are more flexible. However, effective modification of a valid contract is still a burdensome project. Modification at Common Law Rule/Preexisting Duty Rule: At common law, a promise to increase compensation under an existing contract is an unenforceable modification to an existing contract because there is no consideration offered for the modification. A promisor cannot provide consideration where that consideration is a duty the promisor already obligated to perform. This is known as the pre-existing duty rule, and preexisting duties are not deemed consideration because the promisor must already perform that duty on the basis of the original contract. Rule/Exceptions to the Preexisting Duty Rule: (1) Mutual Modification: (a) A promise to increase compensation under an existing contract is enforceable as a mutual modification to the contract if: (i) both parties agree to a performance that is different from the one required by the original contract; (ii) the difference in performance is not a mere pretense of a newly formed bargain [e.g., Midway through the voyage, the sailors threaten to cease work unless they are promised an additional $50 each. The Captain reluctantly agrees, and in exchange, the sailors promised to wake him each morning with a chorus of "Rule Britannia." Although there appears on the surface to be a "mutual modification: of the original contract, it is a "mere pretense" of a new bargain and, accordingly, the promised increase in compensation is unenforceable under the preexisting duty rule]. (2) Unforeseen Circumstances: (a) Where a promise of increased compensation is given in exchange for a performance, and that performance is rendered substantially more burdensome than reasonably anticipated by the parties when they entered the contract, then the preexisting duty rule will not apply [e.g., same basic facts as in the previous example, but the fishing nets provided by the captain turn out to be defective and thus increase the workload of the sailors in a manner substantially in excess of what was reasonably contemplated under the original contract. The sailors threaten to cease the fishing unless they are promised an additional $50 each, and the captain reluctantly agrees. The promise is enforceable despite the preexisting duty rule in view of circumstances not reasonably anticipated by the parties at the time of contracting]. Modification and the Sales of Goods under the UCC Rule/Good Faith Test: (a) Under the UCC, the preexisting duty rule is abolished and an agreement modifying an existing contract for the sale of goods needs no consideration to be binding. Modifications must, however, meet the UCC's good faith test, and failure to do so will render them unenforceable. (b) The good faith test for modifications applies even to modifications that are supported by consideration. Similar to contemporary common law rule that requires more than mere pretense of a new bargain to secure enforcement of mutual modification, a bargained-for-modification is unenforceable under the UCC if the appearance of the mutual bargain is merely a pretext to hide a bad faith change of terms. Rule/Duress and Midterm Modifications: (a) In addition to challenging a midterm modification under the rules just discussed, a party who agrees to a contractual modification in commercially extortionate circumstances may also be able to raise the defense of duress. (b) The duress defense is available not only in common law cases, but also in sales of goods cases governed by the UCC. Rule/Effect of "No Oral Modifications Clauses:" (a) Unless required by the SOF, modifications can generally be oral or written. HOWEVER, the enforceability of an oral modification to an agreement may depend upon whether the contract contain a "no oral modifications" clause. (1) There is no specific language required for this type of provision. "No oral modifications," all modifications must be in writing," or any such similar language will suffice. (b) Common Law Cases: (1) Originally, there was a common law rule that made oral modification clauses invalid, and even today most courts will refuse to enforce them where a party has reasonably relied on the oral agreement at issue. HOWEVER, their enforcement is becoming more likely in modern cases, particularly in the construction context. (c) UCC Cases: (1) clauses prohibiting subsequent oral modifications are presumptively valid; (2) an oral modification made in violation of such a clause may nevertheless be enforceable if the disadvantaged party relies on the modification or the parties perform in accordance therewith.
Performance, Modification, and Excuse 1. Obligations under the UCC
Rule/In General: The seller's obligation is to transfer and deliver, and the buyer's obligation is to accept an pay in accordance with the contract. Seller's Obligations: Rule/Seller's Obligations/Non-carrier Cases: (1) Non-carrier cases are contracts in which it appears that the parties do not intend for the goods to be moved by common carrier. (2) In non-carrier cases, the seller has an obligation to tender delivery-that is, to put and hold conforming goods at the buyer's disposition and give the buyer any notification reasonably necessary to enable him to take delivery; (3) tender must be at a reasonable hour, and the goods must be kept available for the period reasonably necessary to enable the buyer to take possession. (a) HOWEVER, unless otherwise agreed upon, the buyer must furnish facilities reasonably suited to receiving the goods. Rule/Carrier Cases: Carrier cases are contracts in which, due to express terms or due to the circumstances, it appears that the parties intend for the goods to be moved by common carrier [e.g., common carrier may include such means as freight, train, boat, or parcel delivery service]. Rule/Carrier Cases/Shipment Contract: Absent some other agreement, there is a presumption that the contract is a shipment contract, under which the seller is not obligated to deliver at a named destination, and bear the concurrent risk of loss until arrival, unless he has specifically agreed to do so or the commercial understanding of the terms used by the parties contemplates such delivery. (a) under a shipment contract, the seller need only put the goods in possession of a carrier and make appropriate arrangements for them to be sent to the buyer, provide the buyer with any document necessary to enable him to obtain the goods, and promptly notify the buyer that the goods have been shipped. (b) The seller's failure to notify the buyer of the shipment or to make a proper contract for the shipment is only grounds for rejection if material delay or loss ensues. Rule/Carrier Cases/Destination Contract: In a destination contract, the seller has agreed to tender the goods at a particular destination. (a) if the contract requires the seller to tender delivery of the goods at a particular destination, the seller must, at the destination, put and hold conforming goods at the buyer's disposition; (b) the seller must also give the buyer any notice of tender that is reasonably necessary and provide the buyer with any documents of tittle necessary to obtain delivery. Tender of documents through ordinary banking channels is sufficient. Rule/Carrier Cases/Free on Board: In contracts that specify that delivery is free on board (F.O.B.) the F.O.B. point is the delivery point. (a) if the contract is F.O.B. the seller's place of shipment, the seller need only, at his expense and risk, put the goods in possession of the carrier [e.g., seller is a shirt manufacturer with a factory in Baltimore. He contract to sell $2,000 shirts to Buyer, who runs a retail clothing store in Las Vegas. If their contract states that the goods are "F.O.B. Baltimore", Seller's risk and expense would end once he put it in possession of the carrier [e.g., the Quick-Ship Co.]. (b) if the contract is F.O.B. the destination, the seller must, at his expense and risk, tender delivery of the goods at the destination location [e.g., if in the above example, the contract states that the goods are "F.O.B. Las Vegas," the seller would continue to bear the expense and risk while the goods are being shipped by the Quick-Ship Co., until the goods are actually tendered for delivery at the destination location]. Rule/Carrier Cases/Free Alongside (F.A.S.): In contracts that specify delivery is free alongside, the seller must deliver the goods alongside the vessel (in manner usual at the port of delivery) or on a dock designated by the buyer an obtain and tender a receipt for the goods. Buyer's Obligations Rule/Payments/Cash & Check: Unless otherwise agreed-upon, the buyer's tender of payment is a condition to the seller's duty to tender and complete delivery. (b) A tender of payment is sufficient when it is made by any means or in any manner current in the ordinary course of business, unless the seller demands payment in cash and dives the buyer a reasonable extension of time to procure it. (c) A buyer's payment by check is conditional and will be defeated if the check is not honored upon presentment. Rule/Inspection of Goods: Generally, unless the parties agree otherwise, the buyer has a right to inspect goods upon tender or delivery before making payment or acceptance. (2) If the contract requires payment before inspection, nonconformity of the goods will not excuse the buyer from making the payment unless: (a) the nonconformity appears without inspection [i.e., it is evident merely from taking delivery]; or (b) despite tender of the required documents, the documents are forged or materially fraudulent such that an injunction against the issuer would be satisfied; (3) Note that payment before inspection will constitute an acceptance of the goods or impair the buyer's right to inspect or any of his remedies. Risk of Loss Rule/Seller Authorized to Ship by Carrier: If the seller is required or authorized to ship the goods by carrier the risk of loss passes to the buyer: (1) when the goods are delivered to the carrier if the contract does not require the seller to deliver the goods at a particular destination (i.e., under a shipment contract, which is the default presumption in carrier cases); or (2) when the goods are tendered at a particular destination by the carrier so that the buyer is able take delivery if the contract requires the seller to deliver the goods at a particular destination. Rule/Goods held by Bailee: If the goods are held by a bailee to be delivered without being moved, the risk of loss passes to the buyer: (1) when the buyer receives a negotiable document of title covering the goods; (2) when the bailee acknowledges the buyer's right to possess the goods; or (3) after the buyer receives a nonnegotiable document of title or other written direction to deliver the goods to the buyer, once the buyer had a reasonable time to present the document or direction to the bailee. Rule/In Any Other Case: (1) if the seller is a merchant, the risk of loss passes to the buyer when the buyer receives [i.e., takes physical possession of the goods); and (2) if the seller is not a merchant, the risk of loss passes to the buyer upon tender of delivery. Effect of Breach on Risk of Loss: (1) If the seller breaches the contract by making nonconforming tender of delivery, the risk of loss remains on the seller until cure or acceptance; (2) if the buyer rightfully revokes acceptance (as described below), the buyer may-to the extent of any deficiency in his effective insurance coverage-treat the risk of loss as having been on he seller from the beginning; (3) if the buyer breaches before the risk of loss passes to the buyer, the seller may-to the extent of any deficiency in his effective insurance coverage-treat the risk of loss as resting on the buyer for a commercially reasonable time.
Gap-Fillers, Interpretation, and the Parol Evidence Rule 3. Interpreting Ambiguous Language
Rule/Interpreting Ambiguous Language: Where the parties did not fail to address an issue, but instead drafted the contract's language in such a way that the language could be subject to more than on interpretation, then a different type of analysis is needed in order to for the courts to properly determine the meaning. Rule/Interpreting Ambiguous Language/Objective vs. Subjective Meaning: The objectively reasonable meaning of a term at the time of contracting would control over a contrary subjective understanding of the term by one of the parties [e.g., Contractor and Homeowner enter a contract for renovations to Homeowner's home. The contract incorporates by reference "the specifications dated 1/11/07," which had been prepared by the homeowner and were attached to the contract before signing. Contractor mistakenly thinks that the specifications referenced were an earlier version prepared by Contractor, and he signs the contract on that understanding. The specifications prepared by Homeowner and referenced in the contract are binding on both parties]. EXCEPTIONS: (a) where one party harbors a subjective understanding of a term that differs from the objectively reasonable meaning, typically the objective understanding will prevail. HOWEVER, if the other party knows or has reason to know of the first party's subjective understanding, then that subjective interpretation will control. (b) Where the evidence demonstrates that both parties shared the subjective understanding of the term at the time of contracting, the mutual subjective understanding will control [e.g., A general contractor is building a bridge. He hires a paving subcontractor to do work at a price to be calculated in accordance with the dimensions of the "concrete surface of the bridge deck". At the time of contracting, both of the parties understand that the concrete work included "the sides and bottom as well as the surface" of the bridge deck and, accordingly, that interpretation would be the controlling interpretation]. Rule/Interpreting Ambiguous Language/Contra Proferentem and the Doctrine of Reasonable Expectations: (1) If an ambiguous term is included in the contract, then it will be interpreted against the party who supplied the term during negotiations or drafting. Thus if more than one meaning is possible, then the meaning more favorable to other party will control. (a) This rule of construction applies to contracts between bargaining equals as well as to contracts of adhesion, which are standard form (take-it-or-leave-it) contracts that a party with superior bargaining power uniformly imposes on its trading partners. Rule/Doctrine of Reasonable Expectations: even unambiguous terms may be interpreted against the drafting party if they conflict with the reasonable expectations of the other party. (a) Most courts will apply the doctrine of reasonable expectations to insurance contracts, but its application is potentially available to all contracts. (b) under the doctrine, courts make a distinction between the dickered terms, which discuss the transaction-specific terms negotiated by the parties, and boilerplate terms, or those terms appearing in fine print. The court will apply the doctrine only to boilerplate terms [e.g., In an insurance contract, the coverage, deductibles, premium amounts, and policy periods are typically the dickered terms. The rest of the agreement, including specification of the claims process and exclusions, are typically set forth in boilerplate and are therefore subject to the doctrine of reasonable expectations]; (c) under this doctrine, the parties' obligations consist of the dickered terms and only the boilerplate terms that would be consistent with the reasonable expectations of the purchaser. Unreasonable boilerplate terms are unenforceable.
Defenses 2. Misrepresentation
Rule/Misrepresentation: Misrepresentation can play two roles in contracts cases: it may be a "sword," as the basis for affirmative relief, or a "shield" a defense to enforcement. (a) Misrepresentation can be used as a basis for affirmative relief under a tort claim brought by the aggrieved party in the form of an action for rescission and damages. (b) Misrepresentations can be used as a defense to an effort to enforce the contract. (2) Misrepresentations are untrue statements or assertions that related to existing facts. (a) misrepresentations relate to existing facts, not to future conduct or action. If one says, "I'm going to do something," and doesn't do it, that's a broken promise, not a misrepresentation. But, if one says, "I'm going to do X," and at the time he makes that promise he actually doesn't intend to do X, that may be actionable as promissory fraud. (b) Misrepresentations must relate to facts, not opinions. A statement that is based on a party's mere opinion, guess or supposition will not be considered a misrepresentation. (i) where one party disguises fact as an opinion, then will constitute a misrepresentation [e.g., an automobile dealer who tells a buyer that he thinks the car runs pretty well, even though he knows that the car does not run well. would be disguising fact as opinion, and thus be making a misrepresentation to the buyer]; (ii) where on party holds himself out to have knowledge or special skill, and asserts an opinion on the basis of the skill or knowledge, then his assertions are held to relate to the underlying facts and, therefore, sufficient, to establish misrepresentation [e.g., a certified mechanic who tells a buyer that he thinks the car runs pretty well, even though he knows the car does not run well, would, in effect, be asserting facts about the state of the car and, thus, making a misrepresentation to the buyer]. Rule/Misrepresentation/Fraudulent Misrepresentation: To establish fraudulent misrepresentation the following must be proved: (a) The defendant must have made an assertion that was inconsistent with existing facts: (1) Most claims of fraudulent misrepresentation are based on oral or written statements. (i) statements that are "half-truths" can be misrepresentations as well [e.g., An advertisement for real estate "suitable for commercial development," where applicable zoning regulations would prohibit such use]. (2)Misrepresentations can also be made through conduct [e.g., when a payor issues a check for a specified amount despite insufficient funds, he is representing that he has finds on deposit to cover the instrument. (a) Active efforts to cover up the truth (termed fraudulent concealment) likewise constitute misrepresentations [e.g., a seller of real estate undertakes efforts to hide termite damage from potential buyers]. (b) The state-of-mind element requires that there was both: (1) Scienter- The scienter requirement relates to the defendant's state of mind with respect to the assertion. The defendant will satisfy the scienter requirement if either (i) he made the assertion knowing it to be false; or (ii) he made the assertion knowing that he had no idea whether it to be true or false; and (2) intent to mislead - The defendant will meet the requirement for intent to mislead if (i) he made the assertion for the purpose of misleading the aggrieved party; or (ii) he made the assertion knowing there was a substantial likelihood that the aggrieved party would be mislead. (c) the misrepresentations must be material to the contract; and materiality can be shown by either an objective standard or a subjective standard: (i) under an objective standard, a misrepresentation is material where such an assertion was likely to induce a reasonable person to enter into the contract [e.g., an assertion that a used car has been carefully inspected by an independent auto mechanic would appeal to a reasonable person and, thus, satisfy the objective materiality test]. (ii) A misrepresentation is material under a subjective standard if assertion of such a fact was likely, for reasons known to the defendant, to induce the aggrieved party specifically into entering the contract [e.g., An assertion that the car was Beyoncé would be material if made to an individual who the seller knew was a Beyoncé fan]. (d) There must have been reasonable reliance on the representation. (i) the standard for determining whether an aggrieved party's reliance on the defendant's misrepresentation is reasonable is whether, under all of the same circumstances, a reasonable person would have relied on the assertion [e.g., (1) reliance would be unreasonable if the aggrieved party has independent knowledge or reason to know that the perpetrator's statement is false; (2) reliance would be unreasonable if no reasonable person would have believed the assertion; (3) reliance would be unreasonable if the aggrieved party could have easily ascertained the truth by cursory inspection of the goods]. (ii) because of the "duty to read," it is generally not reasonable to rely on a trading partner's characterization of a writing's content. However, two situations in which reliance on the contents of a writing would be reasonable are: (a) if the defendant is a merchant and the aggrieved party is a customer; or (b) if the defendant induces the aggrieved party not to read the writing by some devious means [e.g., the perpetrator induces the victim to sign a contact that the perpetrator knows to contain terms of which the victim is unaware and in circumstances in which the perpetrator know the victim won't have time to read the contract]. Rule/Misrepresentation/Non-Fraudulent Misrepresentation: (1) Non-fraudulent misrepresentation may either be negligent or innocent misrepresentation. For either claim, the following elements must be established: (a) the defendant must have made an assertion that was inconsistent with existing facts; (b) the misrepresentation must be material to the content; and (c) there must have been reasonable reliance on the misrepresentation. (2) Because negligent or innocent misrepresentation is not intentional, there is no need to make a showing of the defendant's state of mind. Instead, the following factors are required: (a) For negligent misrepresentation, the aggrieved party must show that the perpetrator would have known that the assertion was false had he exercised reasonable care; (b) For innocent misrepresentation, the aggrieved party need only show that the perpetrator made an assertion not in accord with existing fact. (3) The same remedies are available to the victim of either negligent or innocent misrepresentation, so the difference between them is merely descriptive but has no further legal consequence. Rule/Misrepresentation/Fraudulent Nondisclosure: (1) The wrong that must be committed under a claim of fraudulent nondisclosure relates not to an assertion made by the defendant but to the defendant's silence where the was a duty to disclose; (2) To prove fraudulent nondisclosure, one must show that: (a) the nondisclosure was material to the contract, under either an objective standard or a subjective standard; and (b) there was reasonable reliance on the nondisclosure. Because fraudulent nondisclosure is based on the defendant's silence rather than an assertion, there is not requirement that the defendant make an assertion. (3) Although there is generally no duty of disclosure to trading partners, if a party is aware of material facts that are unlikely to be discovered in the exercise of ordinary car and diligence, then there will be a duty to disclose that information in these circumstances: (a) where the parties enjoy a relationship of trust and confidence [E.g., family members or with a client]; (b) if a party has made an assertion that was true at the time but has been rendered untrue by intervening events [e.g., if a person make an assertion that an individual plans to keep the property in an undeveloped state and later learns that the individual no longer intended to keep the property undeveloped, he would need to disclose this change]; (c) if the obligation of good faith would require that the party disclose the information [e.g., a seller of residential real estate who fails to disclose a termite infestation to the buyer would have a good faith obligation to disclose the infestation to the buyer]. Rule/Misrepresentation/Remedies: (a) Victims of fraudulent misrepresentation, non-fraudulent misrepresentation, and fraudulent nondisclosure may use those claims in two ways: (1) Avoidance - all three claims give the victim the power of avoidance which may be exercised to defend against a breach of contract claim brought by the misrepresenting party; or (2) Rescission - all three claims enable the victim to bring a tort action to rescind the contract and collect damages for reliance on the misrepresentation, including consequential damages. (b) Victims of fraudulent and non-fraudulent misrepresentation have a third option: they may live with the contract and sue for the benefit of the bargain. (1) The victim of fraudulent misrepresentation may bring a tort action seeking the benefit of the bargain. This would allow the victim to recover the difference between the value of goods or services actually received and the value of the promised goods or services. This tort remedy is the functional equivalent of expectations damages. (2) This remedy of bringing a tort action for the benefit of the bargain is not available for claims of negligent or innocent misrepresentation. However, parties bringing such claims may be able to characterize the assertion contrary to fact as a warranty and thus obtain expectation damages by asserting breach thereof. (c) Because of fraudulent misrepresentation in an intentional tort, the aggrieved party may secure punitive damages against the defendant to penalize the defendant on the basis of the fraudulent intention. (1) Punitive damages are not available for claims of negligent or innocent misrepresentation or fraudulent disclosure.
Remedies 3. Other Possible Remedies
Rule/Other Possible Remedies/Promissory Estoppel: (a) The type of interest a party may recover under a claim of promissory estoppel depends on the jurisdiction in which it brings suit. Courts are split: (1) some courts award expectation damages; (2) other courts, instead, favor reliance damages; and (3) other courts award damages on a case-by-case basis and tailor the remedy to the injustice at issue, where possible. (a) In doing so, the court will focus on: (1) the strength of the proof for each of the individual elements of the claim; (2) the blameworthiness of the breach; (3) the extent of the detrimental reliance by the aggrieved party; (4) the relative positions of the parties; and (5) the availability of alternative options to granting full enforcement of the promise. Rule/Other Possible Remedies/Restitution and Unjust Enrichment: (a) A party who bestows benefits on another may seek to recover the value of those benefits in action for restitution. A party aggrieved by a breach of contract may have the option of electing a restitutionary rather than expectation, recovery. However, there are may other contexts in contract law in which a party may be entitled to restitution pursuant to the policy of avoiding unjust enrichment. (b) The following are contexts where restitution might be available: (1) Benefits Conferred Under a Contract - (a) When a party bestows benefits on his trading partner in connection with what turns out to be a "failed" contract (e.g., unenforceable because of the SOF, or a failed condition, or voidable because of incapacity, fraud, duress, undue influence, or unconscionability), the party bestowing the benefits may ordinarily recover their value via restitution, subject to offset by any benefits received from the other party) [e.g., a contract for the sale of land without a writing is unenforceable but an aggrieved party may be able to recover the value of improvements made to the land via restitution if seller reneges); (2) Benefits Conferred by a Breaching Party - (a) at common law, the breaching party was not entitled to receive any recovery for the benefits it may have conferred under the contract. The modern rule is that such recovery is possible; (i) the breaching party may recover the amount of benefit he conferred on the aggrieved party less any damages to the aggrieved party as a result of the breach (otherwise known as "the right of offset"); (3) Emergency Benefits Conferred by a Health Care Professional-(a) the general rule is that a person who bestows benefits without a request from the benefiting party is considered a volunteer of officious intermeddler who will not be entitled to any recovery (b) HOWEVER, an exception applies to doctors and other health care professionals who provide emergency health care to a patient who is unable to consent. These health care professionals are entitled to the reasonable value of those services they have rendered, even where those efforts were unsuccessful. This exception does not apply to non-medical persons whose life-saving efforts are deemed gratuitous; (4) Benefits Conferred by Mistake. (a) A party who mistakenly confers benefits to another party may be entitled to restitution. In this situation, the following will be considered: (1) the blameworthiness of the error; (2) whether the recipient was aware of the error in the time to prevent it; and (3) whether the recipient availed himself of the benefits at issue [e.g., After a hurricane, Owner hires Contractor to perform repairs on his storm-damaged home. Because of confusion caused by damage to street signs and mailboxes, Contractor performs the repairs on the wrong house. Contractor is entitled to restitution from the benefiting homeowner if the latter was aware of the Contractor's mistaken efforts and remained silent. Rule/Other Possible Remedies/Agreed to Remedies: (a) Parties to a contract may also contract out of the legal and equitable remedies available under the law, by specifying the agreed-to remedies in the contract. (b) These remedies typically take two forms: liquidated damages provisions and provisions limiting or excluding damages. (c) Provisions limiting or excluding damages: (1) the parties to a contract may limit or exclude the availability of certain damages. Thus, parties can limit their remedial rights to those provided for in the contract or exclude rights to remedies that it would otherwise be entitled to under the law. (2) There are different types of exclusive remedies provisions, including: (a) provisions that limit or alter the measure of damages available; and (b) exclusive remedies provisions, in which the parties expressly agree that only one remedy, or certain specified remedies, will be available in the event of a breach; (3) such provisions are enforceable unless they are unconscionable. Under the UCC, limitation of consequential damages for personal injury in the case of consumer goods is prima facie unconscionable.
Gap-Fillers, Interpretation, and the Parol Evidence Rule 1. In General
Rule/Parties to a contract ordinarily reach explicit and unambiguous agreements on the essential issues of the contract, such as quantity and price. However, they frequently fail to be as explicit as to the details. This can result in either gaps in the contract or contractual language that is subject to more than one interpretation. This section deals with the rules courts use to fill contractual gaps and resolve contractual ambiguities.
Defenses 6. Public Policy
Rule/Public Policy: It is a claim that that courts should not enforce a contract because doing so would violate or undermine some important judicial policy. Public policy may be raised as a defense to the enforcement of a contract in the following four contexts: (a) where the subject matter of the contract itself is specifically prohibited by law; (b) where a contract is formed for the purpose of committing a crime or violating a legal regulation; (c) where the contract performance would not constitute a crime, but would constitute a tort; or (d) where the contract performance would violate certain values and freedoms designated by the state or jurisdiction [e.g., a contract that prohibits one party from marrying for an extended period of time would violate public policy of promoting free and consent-based marriages]. Rule/Public Policy/Sources of Public Policies: (a) legislation - legislation is frequently the source of the policy under the public policy defense; and (b) judicial decision - (1) where the subject matter of the contract is not specifically prohibited under law, then the public policy defense may operate to: (i) fill in gaps where there is no direct legislative warrant on point; and (ii) in most jurisdictions, promote larger notions of the public good. (2) Judicially developed public policies include: (a) policies grounded on moral and social values, such as the policy against impairment of family relationships; (b) policies based on economic considerations, such as the policies against restraints on trade; and (c) policies designed to protect governmental processes and institutions, such as the policy of protecting the integrity of voting elections [e.g., a contract under which a private citizen agrees to vote a certain way in exchange for a year's supply of beer would violate the public policy of free and unrestricted exercise of the right to vote]. Rule/Public Policy/Operation of the Defense to Public Policy: (a) The public policy defense is almost invariably invoked as a defense in an action by one of the parties seeking enforcement of a contract against the other. When it is successful, the defendant will win irrespective of whether he was the party who promised to perform the public policy violation or the other party paying for it; (b) the idea is not that the courts have any special solitude for defendants, but rather that courts will refuse to play any role whatsoever in the enforcement of contracts violating public policy, with the result that the party attempting to enforce the contract will be out of luck. EXCEPTION: An exception to the public policy defense may exist where one of the parties is more egregiously "in the wrong" than the other party. In the language of the courts, in such cases the parties are not in pari delicto (equally at fault), and the more innocent party may be able to secure restitution of any benefits conferred on the guiltier party despite the fact that the contract itself remains unenforceable. (1) The in pari delicto defense is most commonly available when a particular statute has been enacted to protect a particular class of persons or actions [e.g., a debtor may be entitled to restitution of insurance premiums, where the creditor coerced the debtor to purchase credit insurance premiums and there is a statute on point that is designed to protect debtors from such creditor coercion]. (c) A contract that is subject to the defense of public policy is not itself void as a matter of public policy. The defense only operates to threat the contract as voidable at the option of the defending party Rule/Public Policy/Contracts Frequently Falling under the Public Policy Defense: (a) Non-competition Agreements - many employment contracts, as well as contracts for the sale of a business, contain a provision that prevents one party from competing for a certain period of time in a certain area. Where this provision is reasonable, the court will generally enforce it. (i) HOWEVER, where the non-compete provision imposes an unreasonable geographical barrier, duration, or term, then the enforcement of such provision may be considered a violation of the public policy of promoting a citizen's freedom to work; (ii) under the blue pencil rule, most jurisdictions remove the offending portion of the non-competition clause while enforcing the remainder of it; (iii) a minority of jurisdictions will hold that inclusion of such provisions invalidates the entire non-completion clause; (b) Sales of Goods via Bribery - if a contract for the sale of goods was based on bribery by any party, then the public policy defense could be used by the victim of the bribe to make the contract unenforceable [e.g., bribing an agent to secure unfavorable contract terms]; (c) Sales of Goods Intended for Unlawful Use - if a contract s for the sale of goods that the seller knows the buyer intends for an unlawful use, then the public policy defense would be available to defeat an action either by the seller seeking payment or by the buyer seeking delivery; (d) Liability-Limiting Provisions - when the provisions of a contract would limit a party's liability for tortious behavior by restricting the right of the injured party to pursue claims against the reckless or intentional harms caused by the party, the provision would not be upheld, because to do so would violate public policy (a) provisions that limit liability for negligent (rather than reckless or intentional) tortious conduct are generally permissible; and (e) Unlicensed Goods and Services: When a contract is for unlicensed goods or services, then the contract may be rendered enforceable by the public policy defense [e.g., an unlicensed contract could be prevented from recovering payment for his services if the homeowner invokes a public policy defense; because the homeowner is not in pari delicto with the unlicensed contractor, an action by the homeowner against the contractor for shoddy work might survive the public policy defense.
Consideration and Promissory Estoppel 1. Consideration and Bargain)
Rule/Rule/Consideration and Bargain/Pre-Existing Duty: Under the pre-existing duty rule, neither doing nor promising to do that which one is already legally bound to the promisor to do can furnish consideration for a promise. In such case, neither benefit to the promisor nor detriment to the promisee exists. Rule/Consideration and Bargain: The general rule in American Contract Law is that a promise is unenforceable unless it is supported by consideration. The exchange of promise for consideration is called a bargain or bargained-for-exchange. To constitute consideration, a performance or a return promise must be bargained for. A performance or return promise is bargained for if it is sought by the promisor in exchange for his promise and is given by the promisee in exchange for that promise. With respect to the requirement for consideration, the performance may consist of: (a) an act or other promise; or (b) forbearance; or (c) the creation, modification, or destruction of a legal relation. Rule/Consideration and Bargain/Bargain: (a) consideration requires a bargain, but does not require bargaining. Thus, there is no requirement that the parties actually negotiate before reaching a deal. "Take it or leave it" - styled transactions are just as enforceable as those following extensive rounds of negotiation. In other words, the bargain requirement refers to the presence (or absence) of an exchange between the parties, not the process of reaching the deal; (b) In contemporary language, bargain may mean that something of greater value was acquired for lesser value, and implies that the transaction was beneficial to the recipient. For purposes on the consideration doctrine, though, bargain merely indicates an exchange, which could be beneficial or detrimental. Rule/Consideration and Bargain/Gratuitous Promise: However, in some promissory transactions, no exchange is contemplated or the existence of quid pro quo (consideration) is questionable. Many of these situations involve intrafamilial transactions, but they occur in the commercial world as well. The problem arises because the promisee has neither promised nor given anything in return to the promisor, leaving the consideration requirement unsatisfied. A gratuitous promise is generally unenforceable under the doctrine of consideration. Rule/Consideration and Bargain/Insufficient Consideration: (1) A promisor defending against an attempt to enforce a gratuitous promise can state his defense in several ways: (a) the promise is not supported by consideration; (b) there is a want of consideration; or consideration is lacking; or (c) there is legally insufficient consideration. [A promise was never made in exchange for performance] (Note: these terms are not to be confused with failure of consideration). Rule/Consideration and Bargain/Failure of Consideration: A failure of consideration is a claim that the party has not performed in accordance with his promise [breach]. Rule/Consideration and Bargain/Bargained-For-Consideration: (a) the majority of jurisdictions evaluate consideration based on the bargained-for-exchange inquiry. A minority of jurisdictions use the benefit/detriment analysis instead. Under this test, consideration is evaluated based on whether there is a benefit to the promisor or a detriment to the promisee. (1) There are also some jurisdictions that use a hybrid evaluation combining the bargained-for-exchange inquiry with the benefit/detriment analysis to define consideration as requiring promise in exchange for detriment. (2) Whether courts are applying the benefit/detriment test or the hybrid promise in exchange for detriment test, the question of whether a particular performance by the promise constitutes a detriment is answered by the so-called legal detriment test. (a) Under this test, the question is not whether the promisee will actually suffer or end up with some sort of net loss due to the transaction. Rather, the question is whether promisee is doing something he had a legal right not to do or is forgoing some activity in which he had a legal right to engage. Rule/Consideration and Bargain/Inadequacy of Consideration: The modern rule is that courts do no police the equivalence of bargained-for-exchanges via the consideration doctrine. In other words, the supposed inadequacy of consideration is no defense to a breach of contract claim. [e.g., if A promises his birthright for a mess of pottage from B, B has nonetheless provided consideration for the promised birthright no matter how meager the value of pottage; EXCEPTION: If A promised to sell blackacre to B for a meager sum and then refused to deliver, some courts would refuse to grant B specific performance because of inadequate consideration, and instead, limit B's remedy to money damages [assuming not enough consideration has been paid]. Modern courts do not police the fairness of bargains via the consideration doctrine. HOWEVER, a party may be able to defeat the enforcement of an excessively one-sided bargain through the defense of Unconscionability. Rule/Consideration and Bargain/Illusory Promises: A promise to perform that leaves performance to discretion of the promising party is an illusory promise and won't constitute consideration. [e.g., A says to B, "I'll sell you as many widgets as you want to order within the next two weeks for $5 a piece" and B agrees to buy that price as many widgets as he decides to order from A, B's promise is illusory and will not constitute consideration for A's promise. Rule/Consideration and Bargain/Gratuitous Promises/Executed Gifts Distinguished: (1) Although gratuitous promises are unenforceable under the consideration doctrine, gratuitous transfers are legally binding upon satisfaction of the requirements of a gift. (2)In most states, delivery of the would-be-gift constitutes a legally binding gratuitous transfer [e.g., if John promises Kristen a new car for her b-day, that would constitute a gratuitous promise and would be unenforceable if John declines to follow through. However, if John gives Kristen a new car for her b-day, then the transfer is legally binding, and John cannot later change his mind and take back the car. Rule/Consideration and Bargain/Bargains Distinguished: Certain considerations will determine whether an exchange is a bargain or a gratuitous promise. (a) Recitals of Consideration: (1) The requirement for consideration is not satisfied by a false recital. EXCEPTION: some courts will enforce option contracts on the basis of a signed writing with a false recital of consideration; (2) A condition on a gratuitous promise also does not satisfy the requirement of consideration. A condition is something a promisee must do to avail himself of the promisor's benevolence [e.g., A invites S, to come live in his guest house without charge so she'll have a safe place to live. S will not be able to advantage of A's offer unless she moves her family 60 miles to A's farm. However, the move is merely a condition of the gratuitous promise, not consideration]. (3) Distinguishing consideration from a condition on a gratuitous promise can be accomplished using three factors: (a) the language of the parties; (i) words suggesting benevolence rather than self-interest, such as "gift," may indicate a gratuitous promises that has a condition rather than consideration; (b) commercial versus charitable or familial context, (i) bargains are standard operating procedure in the commercial context, and gratuitous promises are rare. By contrast, gratuitous promises are made in the family and charitable context more regularly; and (c) the benefit to the promisor; (i) in case of doubt, the most important factor in determining whether a particular performance is consideration or a mere condition on a gratuitous promise is whether the promisor benefits from the performance. Rule/Rule/Consideration and Bargain/Past or Moral Consideration: There are two sets of circumstances in which a promisee may be able to enforce a promise that is not supported by consideration: (1) past or moral consideration and (2) promissory estoppel Rule/Rule/Consideration and Bargain/Past or Moral Consideration: (a) As a general rule, a promise given in exchange for something already performed will not satisfy the bargain requirement [e.g., the court refused to enforce the father's after-the-fact promise to compensate a Good Samaritan for nursing his dying son, and the case would likely come out the same today, Mills v. Wyman, 20 Mass. (3 Pick.) 207 (1825)]. (b) EXEPTIONS: (1) a written promise to pay a debt barred by the SOL is binding; and (2) a written promise to pay all or part of an indebtedness that has been discharged by BK is binding. (c) Minority: a minority of jurisdictions, supported by Section 86 of the Second Restatement, will enforce a promise made in recognition of a past benefit conferred so long as: (1) the promisee conferred the benefit on the promisor and not on a third party; and (2) the benefit is material [e.g. (1) A sees B is in grave danger and heroically intervenes to save the latter, injuring himself in the process. B gratefully promises to compensate A for his efforts, the court enforced the promise because A's efforts bestowed a material benefit on B, Webb v. McGown, 168 U.S. 196 (Ala. Ct. App. 1935); e.g. (2) Because the Good Samaritan in Mill v. Wyman bestowed the benefit of nursing services on the promisor's son rather than on the promisor, the promise would not be enforceable. EXCEPTION: where the promisor makes a promise in recognition of benefits that the promisor received under the terms of a contact, the promise is not enforceable (e.g., employer promises a retiring employee a pension worth half the latter's salary. Although the employee's past work no doubt bestowed material benefits on the employer, the work was done under contract (i.e., EE was paid under the terms of the employment agreement), and accordingly, the ER's promise is not enforceable). Rule/Consideration and Bargain/Promissory Estoppel: (a) A promisee that reasonably relies to his detriment on a gratuitous promise may be able to enforce that promise. The doctrine of promissory estoppel allows for the enforcement of certain promises even where there is no consideration in return. (c) There are four requirements that must be met in order for promissory estoppel to be available: (1) a promise: (a) clearly, in order for promissory estoppel to be available, there must be a promise on which the claim is based. Statements of present intention or vague statements about the future would not qualify as a promise. The statement must contain an unambiguous assurance of future action to be considered a promise; (2) foreseeable reliance: (a) the promisee's reliance must be reasonably foreseeable to the promisor at the time of the promise; (3) actual reliance: (a) the actual reliance must be induced by the promise. This means that the action or reliance cannot be taken account of other factors; and (4) injustice without enforcement: (a) a promise that meets the other elements required by promissory estoppel will be binding if injustice can be avoided only by the enforcement of the promise; (b) although this requirement may sound vague and open-ended, most courts take it seriously and are willing to use promissory estoppel to depart from the general rule that gratuitous promises aren't enforceable only where the prospect of injustice is convincingly established; (c) there are several factors that a court will analyze: (i) The court will consider the strength of the case as a whole, and where there is substantial proof of the other elements, the claim that injustice cannot otherwise be prevented is strengthened. Alternatively, where there is weak evidence of the other elements, it is less likely that the court will find that injustice could not otherwise be prevented; (ii) The court will consider the blameworthiness of the breach. Thus, a willful breach provides a stronger case for promissory estoppel than a breach occasioned by circumstances beyond the promisor's control; (iii) the court will consider the balance of equities between the parties. Thus, the requisite injustice is easier to establish the greater the harm of non-enforcement to the promisee is, and more difficult to establish the greater the harm of enforcement to the promisor is [e.g., An uncle's promise to give his nephew $10,000 upon graduation from a specific school is more likely to be enforced if the nephew turned down scholarship opportunities at other schools in order to attend the school of the uncle's choosing, or if the school would have been out of the nephew's reach but for the uncle's promise to pay. It is less likely that a court would enforce and uncle's promise to pay if the nephew has a lucrative job and the uncle's family has in the meantime suffered serious financial setbacks since making the promise); (iv) the more detrimental a promisee's reliance, the easier it is to establish the requisite injustice. Some courts treat detrimental reliance as independent requirement to be established in order to prove promissory estoppel [e.g., an uncle promised to pay nephew $100,000, upon graduation from law school. The nephew in reliance on the promise decided to borrow his way through school, giving up a chance to attend a lesser-ranked, less expensive school. In this case, it is very possible that the nephew's job prospects and earning potential upon graduation from the high-ranked school are dramatically improved, and a court might find that his reliance on his uncle's promise was, in fact, beneficial and not detrimental. As a result this would prevent the promise from being enforced; (v) some courts treat enforcement of a gratuitous promise as a last resort. In those jurisdictions, enforcement via promissory estoppel is barred where other alternatives that could prevent injustice exist. HOWEVER, all courts would find the existence of options short of enforcement of the promise to be a highly relevant factor in deciding the nature of the injustice at risk
SOF 4. Analyzing Problems Under SOF M Y L E G S
Rule/Specific Coverage under the Common Law SOF: (a) In most states, six categories of agreements are governed by the SOF: Marriage, Land, Year, Executor, Guarantee, Sale of Goods. (b) Individual sates have added additional categories to the six covenant conventional ones. Two common additional categories are real estate brokerage agreements and non-marital cohabitation agreements. Rule/Promise to Marry: Most states have abolished the cause of action for breach of a promise to marry, though it still exists in some jurisdictions. In jurisdictions that recognize the claim, the coverage of the SOF has been narrowed as follows: (a) a promise to marry that has been given in exchange for dowry or other settlement is governed by the SOF, and there must be a signed writing in order to secure enforcement of either party's obligation; (b) an exchange of promises to marry is not governed by the SOF and does not need signed writing to secure enforcement. Rule/Performance within One Year: (1) The year at issue under the one-year provision is measured form the date of the contract's formation rather than the date of the beginning of the performance. A contract that contemplates a duration of less than a year performance may nonetheless fall under the SOF if performance is not to be completed until more than one year after the contract's formation [e.g., In June of her first year of law school, S enters an oral agreement with Law Firm to work for the firm during June, July, and August of her second summer. Although the duration of the contemplated performance is only three months, the performance will not be complete until 14 mos. after the making of the agreement. Accordingly, the contract is governed by the one-year provision, and a signed writing is required for enforcement]. (2) When a contract does not specify a date by which performance is to be completion, the question of whether a particular contract is to be performed within one year of the making thereof is answered by determining whether it is at all possible to complete the required performance within a year's time [e.g., (a) It is highly unlikely that the construction of an oil-producing facility in a war-ravaged country would be completed within a year. But because the performance is possible within that time - even if exceedingly unlikely - the agreement is not governed by the one-year provision, and no signed writing is required; (b) One of the parties to an oral construction agreement attempts to back out 14 months into the project and raises the SOF as a defense. Because the prospect of performance is measured from the point of making the contract (when completion within one year was possible) rather than from the time of the dispute (when completion within the first year is clearly no longer possible), the agreement is not governed by the one-year provision and no signed writing is required; (c)A three-year exclusive representation agreement between a professional athlete and his agent cannot be performed within a year, no matter how diligently the agent works. Accordingly, the agreement is governed by the one-year provision, and a signed writing is required. (3) virtually any contract can be breached or excused within a year of its formation. However, courts typically hold that a breached or excused contract would not be a fully performed contract for purposes of the one-year provision [e.g., Although a two-year contract for an employee's services might be breached by the employee's poor performance at some point during the first year, the contract is nevertheless governed by the one-year provision because full performance will take two years]; (4) Under the majority rule, a lifetime or permanent contract of employment is not governed by the one-year provision because the employee's death is possible within the first year, resulting in termination of the contract without breach. The minority does treat these types of contracts as falling within the one-year provision. Rule/Land-Sale Contracts: (1) the land provision has been modified under some circumstances; (2) the land provision governs contracts for the sale of an interest in land, and most courts take the expression "contract for sale" literally, thus distinguishing between a contract for a future sale and a present conveyance - (a) the contract for future sales is governed by the land provision and requires a signed writing. The present conveyance of land promised for money is held outside the land provision; (3) although real estate brokerage agreements certainly contemplate the sale of an interest in land, most court treat brokerage agreements as falling outside the land provision. The reason is that the brokerage agreement is really service contract and not a contract for real estate. (a some jurisdictions have resolved this by adding a separate category to their SOF specifically for brokerage agreements; (4) Most courts treat a lease of real property as a contract falling within the land provision because a leasehold is an interest in land. Other jurisdictions exclude short-term leases of a year or less from the SOF. Rule/Guaranty/Suretyship Agreements: (1) The general rule that a promise to answer for the debt of a third party - a Suretyship or guaranty agreement-is subject to the SOF has two critical exceptions: (a) When the creditor discharges the original debtor from his obligation on the faith of a guarantee by a third party to pay the debt, these agreements are not governed by the SOF and do not require a signed writing by the guarantor [person agreeing to pay the debt]; (b) under the main purpose exception, if the main purpose of the guarantor's promise is to protect or promote his own economic interests, rather than the interests of the debtor, then the agreement is not within the SOF and no signed writing is required; (1) the mere presence of a selfish interest is not sufficient to trigger this exception, it must be central to the promise. Rule/Satisfaction of the SOF: If a contract falls within the SOF, then the general rules is that the contract is unenforceable unless evidenced by a writing signed by the party against whom enforcement is sought. In essence, there are two requirements for satisfaction of the SOF: (1) a writing; and (2) that the writing be signed. Rule/The Writing Requirement: There is no requirement that the parties put their actual agreement in writing, all that is necessary is that the writing be a memorandum thereof, which can be prepared before, during, or after contract formation. Under the common law, the following memoranda have been held to satisfy the writing requirement: (a) a letter from one of the parties to a third party describing the agreement; (b) the written offer, acceptance of which formed he contract; (c) a letter from one of the parties to the other party repudiating, and so admitting, the agreement - (1) the memorandum need not document the transaction in detail. Only the following terms are required: (I) the identity of the parties to the transaction; (ii) the nature and subject matter of the contract; and (iii) the essential terms of the unperformed promises in the agreement. Rule/Required Description in Cases under the Land Provision: the case law is split on the kind of description you need to satisfy the land rule under the SOF, with some cases holding that an address or its equivalent will do the trick and others requiring a full legal description. Rule/Electronic Email: most courts treat electronic documentation, such as e-mail, as satisfying the writing requirement. Rule/The Signature Requirement: (1) the actual signature of the party against whom enforcement is sought is not necessary. Any symbol, including initials: typed, stamped, or preprinted signatures, or letterhead, if used with the intention to authenticate the writing, will suffice. (2) Most states have adopted the Uniform Electronic Transactions Act (UETA), pursuant to which electronic signatures are considered to satisfy any legal writing requirements. Rule/Signature Requirement/Tacking Together Multiple Documents: (a) The writing need not be a single document; a party may satisfy the SOF by tacking together several documents which, once combined, satisfy all the necessary requirements for the SOF. (1) If all the documents are signed by the party against whom the contract is being enforced, or if a singed document incorporates the unsigned documents by reference, then a signature requirement is not satisfied. (2) If unsigned documents are not incorporated by reference in a signed document, "tacking together" the signed and unsigned documents to satisfy the SOF is nevertheless permissible if: (a) there is at least one signed writing unambiguously establishing a contractual relationship between the parties; (b) the signed and unsigned documents clearly refer to the same subject matter; and (c) there is clear and convincing evidence of acquiescence to the unsigned documents by the party against whom enforcement is sought. Rule/Signature Requirement/Performance/land: (1) The SOF may be satisfied with respect to some of the categories governed by contract via part performance. (2) Land Contracts: (a) part performance will make an oral contract for the sale of land enforceable in two situations: (i) in an action by the buyer against the seller, but not in action by the seller against the buyer; and (ii) in an action for specific performance, but not in an action for money damages; (b) part performance requires a showing of any combination, or of all three, of the following: (i) payment of all or part of the purchase price; (ii) taking of possession; and (iii) making substantial improvements to the property. Rule/Signature Requirement/Performance/One-year Contracts: Full performance of an oral contract for services by the party performing the services will make the contract enforceable against the paying party. On the other hand, part performance is not compensable on the contract. HOWEVER, the performing party may be able to recover for the reasonable value of the services actually rendered via quantum meruit. Rule/Signature Requirement/Performance/Sale of Goods Contracts: The UCC SOF may be satisfied by part performance of a sale of goods contract. Rule/Enforcement Where the SOF is Not Satisfied: (a) if the contract is within the SOF but the SOF is not satisfied because the breaching party never signed a written document, the aggrieved party may nevertheless be able to secure some protection for his interests via an action for restitution or promissory estoppel; (b) Recovery for Benefits Conferred: Where one party bestows benefits upon another in connection with an oral contract, even if the enforcement of that contract is barred by the SOF, the aggrieved party may recover something. The party has the option of filing a cause of action for restitution, seeking to recover the value of the benefits conferred. Alternatively, if services are involved, the party may recover on a theory of quantum meruit, meaning he could sue to recover the reasonable value for the services he rendered. (c) Promissory Estoppel - where a party suffers losses in reliance on an oral contract, but the enforcement of that contract is barred under the SOF, the party may be able to recover damages via promissory estoppel [e.g., (i) where a party to an oral contract within the SOF promises the other party that he has created or will create a signed writing evidencing the parties' agreement, and the other party relies on that assurance by failing to take other steps to satisfy the SOF, most courts will find promissory estoppel against the party who falsely made assurances of a signed writing. (ii) In many cases, however, there is no assurance that a writing has been or will be created, and the reliance at stake will instead be based n the very contract whose enforcement is barred by the SOF - and employee who leaves his current job and moves across the country based on an oral two-year employment contract, only to be dismissed before he starts his new job. Invoking a promissory estoppel in this setting is both more controversial and more difficult]. (2) Some courts hold that promissory estoppel is available to protect reliance on a contract whose enforcement is barred by the SOF. Other courts have held that the common law promissory estoppel doctrine is preempted by the SOF. (3) Among those courts that do recognize promissory estoppel, some will impose stiffer requirements on the claimant than those applicable under an ordinary promissory estoppel action. In particular courts will look for: (a) the definite and substantial character of reliance, and its relationship to the remedy sought; (b) the extent to which the reliance is corroborated by the evidence of the formation and terms of the contract; and (c) the extent to which the formation and terms of the contract are otherwise established by clear and convincing evidence.
Assignment of Rights and Delegation of Duties/Delegation of Duties
Rule: A delegation occurs when a third party agrees to satisfy a performance obligation owed by one of the parties to a contract. (A) Obligee = the party for whom the performance is owed (B) Obligor = the party with the performance obligation, and is also the delegator (the party who delegated his performance to a third party; and (C)Delegatee = the party to whom the performance obligation was delegated. Rule: A delegation of duties will not discharge the delegator's; only a novation assented to by the obligee will have this effect. Rule/Novation: If there is a novation, the delegator is relieved from the obligations under the contract. This requires a clear promise by the obligee to release the delegator in return for the liability of the delegatee. simple assent to the delegation is not enough to effectuate a novation, there must also be a promise to release the delegator. Rule/Liability of the Delegatee: When the delegatee has agreed to perform the delegator's contract obligations, he is liable to the delegator if he does not do so. [Under the third-party beneficiary theory, the delegatee is also liable to the obligee, because the obligee is an intended beneficiary of the promise made to the delegator. (e.g., this is a situation where the delegatee is aware that it's performance is for the obligee). Rule/Delegable Duties: Generally, all obligations can be delegated. EXCEPTIONS: (1) When the performance in question is personal and the recipient must rely on qualities such as the character, reputation, taste, skill, or discretion of the party who is to render the performance; or (2) when the contract prohibits delegation (unlike prohibitions against assignments, contract provisions barring delegation are fully enforceable) Rule/Delegable Duties: COMMON LAW - at attempted delegation of non-delegable duties operates as an immediate breach of the contract and give the other party the immediate right to sue. Rule/Delegable Duties: UCC- any delegation of performance may be treated by the other party as creating reasonable grounds for insecurity, and the other party has the right to demand adequate assurances form the assignee without prejudicing his rights to the assignor.
Contract Formation: Offer and Acceptance 1. In General
Rule: An offer is the manifestation of willingness to enter into a bargain, so made as to justify another person in understanding that his assent to that bargain is invited and will conclude it [Restatement Second Contracts Sec. 24].
Gap-Fillers, Interpretation, and the Parol Evidence Rule 4. Trade, Usage, Course of Dealing, and Course of Performance
Rule: Extrinsic evidence of trade usage, course of dealing, or course of performance will be available as an additional means of filing contractual gaps or resolving contractual ambiguities. (a) A usage of "trade" is any practice or method of dealing having such regularity of observance in a place or trade as to justify an expectation that it will be observed with respect to the transaction in question. (b) a "course of dealing" is a pattern of conduct concerning previous transactions between the parties that is fairly to be regarded as establishing a common basis of understanding for interpreting their subsequent expressions and other conduct; (c) a "course of performance" is present when a particular contract involves repeated occasions for performance by a party and the other party, with knowledge of the nature of the performance or acquiesces in it without objection. Rule/Admissibility: Evidence of trade usage, course of dealing, or course of performance is admissible under the UCC and is typically available in common law cases as well. (a) such evidence may be used to supplement the express terms of a contract or to give meaning to a particular ambiguous term. HOWEVER, it is not admissible to contradict the express terms of a contract. EXCEPTION: course of performance evidence may be available to establish the waiver or modification of an express term. (b) In the event of a conflict, course of performance prevails over course of dealing and usage of trade, and course of dealing prevails over trade usage. [Note: Express terms of the K override both course of dealing and usage of trade].
Defenses 7. Mistake
Rule: In all unilateral mistake situations. If a mistake is made in an offer and the offeree is or should be aware of the mistake, there will be no contract.
Gap-Fillers, Interpretation, and the Parol Evidence Rule 5. The Parol Evidence Rule
Rule: When a contract is expressed in a writing that is intended to be the complete and final expression of the rights and duties of the parties, parol evidence of prior oral or written negotiations or agreements of the parties (or of their contemporaneous oral agreements), which varies or contradicts the written contract is not admissible. Rule/When the Parol Evidence Rule Will Not Apply: (1) Subsequent Agreements Rule: The parol evidence rule only applies to oral or written communications made prior to or contemporaneously with the execution of a written agreement. This means that it will not apply to subsequent agreements entered into after the execution of the written agreement. (2) Collateral Agreements Rule: The parol evidence will not affect agreements between the parties that are entirely distinct from the written agreement of the contract at issue. Evidence offered to prove or enforce such "collateral agreements" is admissible. (3) Attacks on the Validity of the Written Agreement Rule: The key predicate to the application of the parol evidence rule is whether there is a valid agreement. As a result, the parol evidence rule will not bar efforts to prove that the written agreement is invalid or unenforceable. The absence of an enforceable agreement may be proved by parol evidence or any other extrinsic evidence. A party can assert such a claim in one of the following ways: failure of a condition precedent; absence of consideration, mistake or duress, fraud or reformation. (a) Failure of a condition Precedent Rule: Evidence that the parties orally agreed to a condition precedent to the contract taking effect as well as further evidence that the condition failed would not be barred by the parol evidence rule where both parties' obligations would be discharged by the failure of that condition. (b) Absence of Consideration Rule: Evidence that a written agreement lacks consideration or that there was a false recital of consideration is not barred by the parol evidence rule (c) Mistake or Duress Rule: Evidence that the written contract was formed from mistake or duress will not be barred by the parol evidence rule. (d) Fraud Rule: The majority rule is that the parol evidence rule will bar extrinsic evidence for fraud. Rule: The minority rule is that extrinsic evidence is inadmissible to prove fraud where the written contract expressly eliminates an element of the claim (e.g., the parties agree that neither party relied in any way on representations made outside the four corners of the agreement). (e) Reformation Rule: The parol evidence is also inapplicable where a party to a written agreement alleges facts entitling him to reformation of the agreement. Reformation is based on the premises that the parties had reached an agreement concerning the instrument, but while reducing their agreement to written form, and as the result of mutual mistake or fraud, some provision or language was omitted from, inserted, or incorrectly stated in the instrument intended to be the expression of the actual agreement of the parties. For the plaintiff to obtain reformation it must be shown: (1) that there was an antecedent valid agreement; (2) which is incorrectly reflected in the writing (e.g., by mistake); and (3) proof of these elements is established by clear and convincing evidence.
Gap-Fillers, Interpretation, and the Parol Evidence Rule 2. Filling in Gaps with Default Provisions
Rule: one of the principal ways that contract law deals with gaps in a contract is by supplying the terms that govern areas where the contract is silent. These provisions can be overridden where the parties provide for a different procedure, but if they do not, the default contract terms will supplement their contract. Standard rules automatically supply contract terms unless they are set aside by party action or agreement. UCC Default Rules for the Sales of Goods/Implied Warranties Rule/Implied Warranties/Warranty of Title and Against Infringement: For the sale of all goods, there is an implied warranty of good tile, of the rightful transfer of the goods, and that no liens or other security interests are attached to those goods; (b) this warranty can only be excluded or modified by: (1) specific language; or (2) circumstances which give the buyer reason to know that the seller does not claim unencumbered title; (c) unless otherwise agreed, a merchant regularly dealing in goods of the kind warrants that they are free of any rightful claims of a third person by way of infringement. HOWEVER, a seller is not liable for such a claim if the buyer provides specifications to the seller and the claim arises out of compliance with the specifications [2-312(3)]. Rule/Implied Warranties/Warranty of Merchantability: If the seller of the goods is a merchant with respect to goods of that kind under the UCC, then there is a warranty of merchantability for the sale of goods. The warranty guarantees that the goods are fit for the ordinary purposes for which those goods would be used [2-314]. Rule/Implied Warranties/Warranty of Fitness for a Particular Purpose: This warranty grants that the goods being sold are fit for the particular purpose for which the buyer intends to use them. The warranty only applies where, at the time of contracting, the seller has good reason to know: (1) the particular purpose for which the goods are required; and (2) that the buyer is relying on the seller's skill or judgment to select or furnish reasonable goods. Rule/Implied Warranties/Excluding or Modifying Implied Warranties of Merchantability and Fitness: (a) the implied warranty of merchantability may be excluded or modified by language mentioning "merchantability" and need not be in writing. However, if it is in a writing, it must be conspicuous; (b) an implied warranty of fitness may be excluded or modified by language in a writing that is conspicuous. Language to exclude all implied warranties of fitness is sufficient if it states, for example, that "There are no warranties to which extend beyond the description on the face hereof. HOWEVER, the above language is not necessary to exclude or modify the warranty if: (1) the contract included an expression like "as is" or "with all faults" other similar language that in common understanding calls the buyer's attention to the exclusion of warranties and makes plain that there is no implied warranty; (2) the buyer has fully examined the goods before entering into the contract, or has refused to do so, in which case there is no implied warranty as to defects that should have been apparent from the examination; or (3) course of dealing, course of performance, or usage of trade indicate that the warranty is excluded or modified. Rule/Express Warranties/Creation: (1) Note that the UCC also provides that a contract can create express warranties. Express warranties by the seller are created as follows: (a) any affirmation of fact or promise: made by the seller to the buyer that relates to the goods and become part of the basis of the bargain creates an express warranty that the goods will conform to the affirmation or promise; (b) any description of the goods: that is made part of the basis of the bargain creates an express warranty that the goods will conform the description; and (c) any sample model: any sample or model that is made part of the basis of the bargain creates an express warranty that the whole of the goods will conform to the same model. Rule/Express Warranties/No Requirement of Special Language: The seller need not use formal words such as "warrant" or "guarantee," nor must the seller have a specific intention to make a warranty in order to create an express warranty. HOWEVER, a mere affirmation of the values of the goods or a statements of the seller's opinion does not create a warranty [2-312]. Rule/Express Warranties/Conflicts with Disclaimer: Words or conduct relevant to the creation of an express warranty and words or conduct tending to negate or limit warranty shall be construed wherever reasonable as consistent with each other. (a) where an express warranty conflicts with a limitation or disclaimer, the express warranty will prevail. Rule/Express Warranties/Measure of Damages: The measure of damages for breach of warranty is the difference at the time and place of acceptance between the value of the goods accepted and the value they would had if they had been warranted [2-714]. Rule/Missing Terms: Under the UCC, where there are pertinent terms missing, the following provisions will supplant the contract: (a) the default rule for a missing "price term" is the reasonable price at the time established by the contract for the delivery of goods; (b) a missing "time term" exists if the contract is silent as to the date of delivery or any other date by which action must be taken under the contract. The default rule for a missing time term is that contractual action must be performed within a reasonable time; (c) the default rule for a missing "place of delivery term: is that the place of delivery will be the seller's place of business unless otherwise agreed. Common Law Default Rules for Service and Employment Contracts: Rule/Common Law Default Rules/Missing Price Term: If one party performs services at the request of another but no priced is discussed in advance, then the default rule under common law will apply. The default rule for a missing prices term is the reasonable value of the services rendered. This is typically available based on quantum meruit. Rule/Common Law Default Rules/Missing Quantity Term: In practically every jurisdiction, the employment-at will rule is the default rule for the duration of an employment contract; absent an agreement to the contrary, an employer may dismiss, and an employee may quit, at any time for any reason. In a majority of jurisdiction, oral, or written assurances of job security made to an individual employee, as well as assurances contained in policy documents distributed to the workforce, may suffice to take the contract out of the default rule [e.g., statements made during a recruiting interview that make assurances of job security would displace the at-will rule. Alternatively, similar written assurances contained in a personal handbook would also displace the rule]. EXCEPTION: Most courts will enforce a clear and conspicuous declaration in a personnel handbook or other employer policy statement to the effect that the employment relationship is "at will." HOWEVER, a conflict between such a disclaimer and other written oral representations by the employer will create a jury question on whether the employer has successfully opted out of the at-will default. The Obligation of Good Faith and Fair Dealing Rule/Good Faith: (a) Another source of gap-filling is the obligation of goods faith and fair dealing. Both the UCC and the Restatement impose an obligation of good faith in the performance and enforcement of contracts. Goods faith is generally defined as "honest" in fact in the conduct or in the transaction concerned" and, in the case of a merchant, good faith means "honesty in fact and the observance of reasonable commercial standards of fair dealing in the trade." (b) the obligation of good faith may only be raised in regards to the performance and the enforcement of contracts, but not to negotiations or other pre-contractual conduct. Challenges to unfair dealing in those settings must be made via claims of fraud, duress, or other defenses, discussed later. (c) The good-faith obligation will also operate to ensure good faith where the terms of the contract leave a critical term, such as price, satisfaction, or quantity, open to the determination of one party. Rule/Good Faith/Open Price Term: if a contract leaves the price to be fixed by one of the parties, then the specified party must fix the price in good faith. Rule/Good Faith/Satisfaction Term: (a) A contract may contain a satisfaction clause, or similar term, whereby he determination as to whether a party's performance obligation is complete is left to the discretion of the other party. In those contracts, the good-faith obligation operates to require the party making the determination to do so in good faith. [e.g. a commercial developer and the seller of a piece of land enter into a contract. The developer must determine in good faith whether or not the future leases are "satisfactory."] Rule/Good Faith/Open Quantity Term: (a) an "output contract:" is a contract in which the buyer agrees to purchase all of a seller's output for a particular goods. (b) a "requirements contract:" is a contract in which the seller agrees to supply the buyer with all of the buyer's requirements for a particular good. (c) under the UCC, the party entitled to determine the particular quantity of goods to be sold - either the buyer demanding the delivery of his requirements or the seller demanding purchase of his output - must make that determination in good faith; (d) in addition to the good-faith requirement, the UCC prohibits any unreasonably disproportionate demand or tender, of there was either: (1) a stated estimate between the parties; or (2) in the absence of a stated estimate, any comparable prior outputs requirement [e.g., the upsidasium market remains stable, but because of a decline in the commercial aviation industry, the demand for Buyer's engines decline precipitously and accordingly Buyer reduces its monthly demand for bearings by 90% in comparison with the previous 24 months. Although Buyer's reduced demand was made in good faith, it is unreasonably disproportionate to its prior requirements and, accordingly, violates 2-306].